Legal Professions Quizes

¡Supera tus tareas y exámenes ahora con Quizwiz!

Zarah is a partner at a law firm. She supervises the work of a junior associate, Charlie. They are representing Whitney in her divorce proceedings. The divorce involves a heated battle over the custody of the daughter of Whitney and her husband, Cody. Zarah asks Whitney whether she knows the password for her husband's e-mail. Whitney gives it to her. Then Zarah gives the password to Charlie and instructs him to copy all the e-mails in Cody's account and read them to see whether Cody is having an affair. Charlie objects. Zarah says that this investigative work could win the case for Whitney and threatens to fire Charlie if he does not comply. Charlie complies. They learn that Cody really is having an affair, and Whitney wins custody. When Cody discovers what happened, he files a disciplinary complaint against Zarah and Charlie. Assuming that Charlie's surreptitious review of Cody's e-mail is unlawful and is a rule violation, which one of the statements is accurate? a. Both Charlie and Zarah are subject to discipline. b. Neither Charlie nor Zarah is subject to discipline. c. Only Zarah is subject to discipline. d. Only Charlie is subject to discipline.

a. Both Charlie and Zarah are subject to discipline. This statement is accurate. Zarah is subject to discipline because Rule 5.1(c)(1) states that a lawyer shall be responsible for another lawyer's misconduct if the lawyer orders the actions. Charlie also is subject to discipline. Under Rule 5.2(b), a subordinate lawyer does not violate a rule if he acts in accordance with a supervisor's reasonable resolution of an arguable question of professional duty, but Zarah told Charlie to engage in unambiguously illegal conduct.

Jill, an attorney, brings a lawsuit on behalf of her client, Ann, against Grant, after Grant fails to repay Ann's loan to him of $5,000. The retainer provides that Jill's contingent fee will be 30 percent of any recovery. Jill files the complaint one day too late, and the suit is dismissed with prejudice. Jill is chagrinned and embarrassed and wants to make amends for her mistake. She would like to pay Ann out of her own pocket without disclosing her error to Ann or to anyone else. She would simply get Ann a cashier's check for the portion of $5,000 that would have been paid to her if the court had ordered Grant to repay the $5,000. May she do so if she avoids making any false statement about the source of the funds? a. No, because Jill is required to tell Ann that she missed the deadline and that the case was dismissed. b. Yes, because this would give Ann the amount she would have expected to obtain as a result of the lawsuit. c. Yes, because she would be doing the right thing without engaging in dishonesty, fraud, deceit, or misrepresentation. d. No, because she is required to report her misconduct to the disciplinary authorities.

a. No, because Jill is required to tell Ann that she missed the deadline and that the case was dismissed. Under MR 1.4, a lawyer must keep a client informed about the status of a matter. The fact that the client's case was dismissed is a significant event in the litigation. A lawyer who makes a serious mistake must inform the client (and should inform her malpractice insurer) of her mistake.

Lawyer Shawn has handled various property and business transactions for Evelyn, her adult daughter Adelaide, and Evelyn's business partner, Oliver, during the last decade. Those matters are all completed. Five years ago, Shawn wrote a will for Evelyn in which she left all of her property to Adelaide. In that work, Shawn represented and gave advice to both Evelyn and Adelaide. Adelaide, who lives with her mother, suffers from disabling epilepsy and has not been able to support herself. After Shawn finished drafting the will, he drafted a letter to Evelyn and Adelaide informing them that his office was closing their file and confirming the termination of their lawyer-client relationship. However, he failed to mail them the letter. Last week, Evelyn came to Shawn to tell him that it's time for Adelaide to stand on her own two feet. She mentioned that Adelaide has become involved with a boyfriend, Felix, and has been staying out nights. She would like Shawn to write a new will for her in which she would leave all of her property to Oliver. She further asks Shawn not to reveal to Adelaide the contents of the new will. May Shawn write the new will for Evelyn? a. No, because he did not obtain Adelaide's informed consent to his representation of Evelyn with respect to the new will. b. Yes, because he will not reveal the contents of the new will to Adelaide without authorization by Evelyn. c. No, because he failed to inform Adelaide that he no longer represents her. d. Yes, because his prior representation of Adelaide was incidental to his representation of Evelyn.

a. No, because he did not obtain Adelaide's informed consent to his representation of Evelyn with respect to the new will. This is the correct answer. Under Rule 1.9(a) Shawn may not represent Evelyn in drafting the new will. Adelaide is a former client of Shawn's. The new matter is the same matter as the one on which he previously represented Adelaide, and Evelyn's interests are materially adverse to Adelaide's, so his revising the will is prohibited unless Adelaide give informed consent. To obtain Adelaide's consent, Shawn needs to reveal confidential information to her about her mother's intentions as to the will. Shawn may not do this unless Evelyn consents to the revelation of those confidences, which she will not.

An attorney represented a local amusement park in a personal injury claim brought by one of its customers. Although the park had a valid defense to the claim, the attorney advised the park to pay the damages requested by the customer in order to avoid bad publicity. Relying on the attorney's advice, the park paid the claim. Is the attorney subject to discipline? a. No, because paying the claim avoided bad publicity. b. Yes, because the park paid the claim although it had a valid defense. c. No, because the park accepted the attorney's advice. d. Yes, because the attorney did not zealously advocate the interests of his client.

a. No, because paying the claim avoided bad publicity. This is the correct answer. When a lawyer gives a client advice, he or she may rely on considerations other than those involved in giving purely technical legal advice. Model Rule 2.1. Consequently, it is proper for a lawyer to rely on moral, economic, and social factors in doing so, which is what the attorney did here when he took potential bad publicity into consideration.

An attorney prepared a will for a client and acted as one of the subscribing witnesses to the client's execution of the will. The will left all of the client's estate to his son. Later, at the client's request, the attorney prepared a second will for the client and acted as one of the subscribing witnesses to the client's execution of the second will. The second will left one-half of the client's estate to his son and the other one-half to his housekeeper. The client died and the housekeeper offered the second will for probate. If the son requests that the attorney represent him in opposing probate of the second will on the grounds of fraud and undue influence, is it proper for the attorney to do so? a. No, because the attorney will be taking a position adverse to a will she prepared and witnessed. b. Yes, because the client's son is a beneficiary under both wills. c. Yes, because after the client's death the attorney may represent his son. d. No, because an attorney guarantees the validity of a will that he or she prepares

a. No, because the attorney will be taking a position adverse to a will she prepared and witnessed. The guiding Rule is MR 1.7, which deals with conflicts of interest. In less the son is able to prove fraud and undue influence, you must assume that the client intended to cut his son's bequest in half. Thus, his son's position is directly adverse to the clients. If the client were alive, MR 1.7(a)(1) would preclude representation of his client by the attorney because the interests of two clients would be directly adverse to each other. Now that the client is deceased, the attorney is still constrained by model rule 1.7(a)(2), which also directs a lawyer not to represent a client if representation of that client may be materially limited by the lawyer's responsibilities to another client or to a third person. (A deceased client is still a client for the purpose of this Rule.) Note also that the attorney will necessarily be called as a witness to the execution of the will and the behavior and circumstances of the deceased client. Because a lawyer may not act as an advocate at a trial in which the lawyer is likely to be a necessary witness on a disputed issue (MR 3.7), the attorney should not agree to represent the son in this will contest.

While representing a client in a personal injury case, the client told the attorney that she had previously killed a man and buried his body in a nearby swamp. The murder is completely unrelated to the personal injury case. The body has not been found, and the client is not a suspect in the crime, which remains unsolved. Must the attorney voluntarily disclose to the authorities his knowledge of the prior murder and the location of the victim's body? a. No, because the information was obtained by the attorney in the course of representation. b. Yes, because the murder remains unsolved. c. No, because the attorney did not represent or advise his client regarding the prior murder. d. Yes, because the attorney is an officer of the court.

a. No, because the information was obtained by the attorney in the course of representation. A lawyer shall not reveal any information relating to the representation of a client unless the client gives informed consent. Model Rule 1.6(a). Here, the information was obtained during the course of the attorney's representation, and consequently "relates" to that representation. Consequently, the attorney should not reveal it.

A defendant was on trial for murder. The victim was killed during a barroom brawl. In the course of closing arguments to the jury, the prosecutor said, "The defendant's whole defense is based on the testimony of his friend, who said that the victim attacked the defendant with a knife before the defendant struck him. No other witness testified to such an attack by the victim. I don't believe the defendant's friend was telling the truth, and I don't think you believe him either." The prosecutor accurately state the testimony, and truly believed the defendant's friend was lying. Was the prosecutor's statement proper? a. No, because the prosecutor asserted his personal opinion about the defendant's credibility. b. Yes, because the prosecutor accurately stated the testimony in the case. c. No, because the prosecutor alluded to the beliefs of the jurors. d. Yes, because the prosecutor, in fact, believed the defendant's friend was lying.

a. No, because the prosecutor asserted his personal opinion about the defendant's credibility. This is the correct answer. At trial, an attorney is not allowed to state his or her personal opinion about the credibility of a witness. Model Rule 3.4.

An attorney was formerly employed by an insurance company as a lawyer who solely handled fire insurance claims. While so employed, she investigated a man's fire loss claim against the insurance company. The attorney is now in private practice. The claim remains unsettled and the man consults the attorney and asks the attorney to represent him for a suit on the claim. Which of the following would be proper for the attorney to do? a. Refuse to discuss the matter with the man. b. Refer the man to an associate in her law firm, provided the attorney does not share in any fee. c. Represent the man d. Refer the man to an associate in her law firm

a. Refuse to discuss the matter with the man. This is the correct answer. "A lawyer who has formerly represented a client in a matter shall not thereafter represent another person in the same or a substantially related matter in which that person's interests are materially adverse to the interests of the former client unless the former client gives informed consent, confirmed in writing." Model Rule 1.9(a). Here, since the attorney formerly represented the insurance company in the same matter, she cannot represent the man. This answer choice is the only choice that reflects this rule.

Three men were indicted for the armed robbery of the cashier of a grocery store. Together, two of them met with an attorney and asked the attorney to represent them. The attorney then interviewed the two of them separately. Each told the attorney that the robbery had been committed by the third man while they sat in the third man's car outside the store. They also said that the third man had said he needed some cigarettes, and that neither knew of the man's plan to rob the cashier. The attorney agreed to the joint representation. One week prior to the trial date, one of the men represented by the attorney told the attorney that he wanted to plea bargain and that he was prepared to turn state's evidence against both of the other two suspects. It is proper for attorney to: a. Request court approval to withdraw as lawyer for both men. b. Continue to represent the man who had not stated that he wanted to plea bargain and, with the other man's consent and court approval, withdraw as his lawyer. c. Continue to represent both men, but not call man who wanted to plea bargain as a witness. d. Continue to represent the man who wanted to plea bargain and, with the other man's consent and court approval, withdraw as that man's lawyer.

a. Request court approval to withdraw as lawyer for both men. A lawyer cannot represent two clients if the representation of one client will be directly adverse to to another. MR 1.7(a)(1). Here, the one client's plan to turn state's evidence against the other client clearly violates this restriction. Importantly, since the common representation has failed (since one client is turning against another), the attorney must ordinarily withdraw from representing all of the clients. MR 1.7, Comment [29].

Kieran, five years out of law school, accepted a job with Sato & Perlmutter, LLP. A couple of months after he started work, Miranda Perlmutter was asked to take over representation of Kasho Natural Foods in an ongoing lawsuit against the Roxbury Box Company. Kasho had fired its previous counsel after a dispute over legal fees. Miranda wants to accept Kasho as a client. However, there may be a problem. Kieran cam to Sato & Perlmutter from another firm, Podkrash and Associates, LLP, where he had worked for three years. The Podkrash firm has been representing the Roxbury Box Company in the Kasho litigation for the last two years. Kieran did not work on the Roxbury matter while he worked at Podkrash. While Kieran was working at Podkrash, he dated Lance, the senior paralegal at the firm. Lance oversaw all the staff work on the Roxbury matter and talked at length about the factual and strategic issues in the case with Kieran on many occasions. For example, the two men talked about a decision of the partner in charge of the litigation that a key document that might impact Roxbury's liability was not covered by a plaintiffs' discover request. Lance disagreed with the partner's decision, but ultimately decided to keep his mouth shut. After consultation with her firm's ethics counsel, Miranda decides to undertake representation of Kasho. However, she does not wish to terminate Kieran's employment as a way to solve any conflicts problem. Instead, she timely screens Kieran from the lawyers working on the matter, sees to it that he receives no part of the fee from the Kasho matter, and provides appropriate written notices to Roxbury about the screening procedures. Was Sato & Perlmutter permitted to undertake representation of Kasho in this manner? a. Yes, because Kieran was properly screened from the Kasho representation. b. No, because this is a situation in which screening is insufficient to avoid the possibility of a conflict of interest so long as Kieran remains at the firm. c. No, because there is a substantial risk that confidential information that Kieran would have learned at the Podkrash firm would materially advance Kasho's position in the litigation. d. Yes, because Lance and Kieran's relationship was personal in nature.

a. Yes, because Kieran was properly screened from the Kasho representation. This is the correct answer. 1.9(b) applies rather than 1.9(a) because Kieran himself did not represent Roxbury; it was a client of his former firm. Rule 1.9(b) would bar Kieran from working on this matter unless Roxbury gave informed consent because (1) the matters are the same or substantially related, (2) the interests of the parties are materially adverse, and (3) Kieran knows confidential information about the former matter that is material to the new matter. However, the firm can undertake work on this matter without Roxbury's consent if the screening procedures specified in that rule are implemented. Rule 1.10 precludes a lawyer in the new firm from handling a matter if another lawyer in the firm would be precluded from doing so, unless the preclusion is based on Rule 1.9(a) or (b) and arises out of the disqualified lawyer's association with a prior firm (both are true in this case). In that instance, the conflict is not imputed to other lawyers in the firm if the firm complies with the screening procedures specified in Rule 1.10(a)(2). Therefore, the Perlmutter firm may go forward only if Kieran does not work on the matter and receives no part of the fee from it, and the firm implements all of the screening procedures required by Rule 1.10(a)(2).

An attorney represents a plaintiff in a tort action against a defendant company. The company sells widgets. The plaintiff's complaint makes a defective product claim seeking recovery of damages. The complaint alleges that plaintiff sustained personal injuries while using the widget as the company intended. Specifically, the complaint asserts that the company's defective design of the widget caused the plaintiff's injuries. During pre-trial discovery, the attorney makes a proper request for documents from the company about the product's design. In reviewing the documents requested, the company's in-house lawyer finds a widget designer's handwritten note stating that, due to the widget's design, the widget could cause the type of injury that plaintiff sustained. The in-house lawyer does not provide the note or refer to it in response to the request for documents or otherwise. Will the in-house lawyer be subject to discipline? a. Yes, because he prevented the plaintiff from obtaining the note. b. Yes, because he has a constitutional duty to disclose the note. c. No, because the note is work product. d. No, because the note is confidential.

a. Yes, because he prevented the plaintiff from obtaining the note. This is the correct answer because MRPC 3.4 prohibits an attorney from unlawfully obstructing another party's access to evidence or conceal something with potential evidentiary value. Here, the lawyer violated the Rule by neither providing the note nor referring to it at all, when the lawyer did not have any lawful basis for preventing the plaintiff's discovery of the note. Generally the attorney may discover any evidence that is relevant to the controversy, provided that the evidence is neither privileged nor work product. Here, the lawyer cannot contend that the law of either attorney-client privilege nor work product protection make lawful his obstruction of the attorney's access to the note and concealment of it.

An attorney representing 10 plaintiffs in a case against a railroad in which the plaintiffs were injured when a train derailed. The railroad offered the attorney a $500,000 lump sum settlement for the 10 plaintiffs. The attorney allocated the $500,000 among the 10 plaintiffs with the amount paid each plaintiff dependent on the nature and extent of that plaintiff's injuries. The attorney reasonably believed the division was fair to each plaintiff. The railroad would not settle any of the claims unless all were settled. The attorney told each plaintiff the total amount the railroad was prepared to pay, the amount that the individual would receive, and the basis on which that amount was calculated. The attorney did not tell any plaintiff the amount to be received by any other plaintiff. The attorney believed that if she revealed to each plaintiff the amount of each individual settlement, it might jeopardize the settlement. Each of the plaintiffs agreed to his or her settlement amount and was satisfied with that amount. Is the attorney subject to discipline for entering into the settlement? a. Yes, because no individual plaintiff knew the amount to be received by any other plaintiff. b. Yes, because the attorney was eating the lawyer for the railroad in making a lump sum settlement. c. No, because the attorney reasonably believed that the division was fair and each plaintiff agreed to his or her settlement and was satisfied with the amount. d. No, because disclosing all settlements to each plaintiff might have jeopardized the entire settlement.

a. Yes, because no individual plaintiff knew the amount to be received by any other plaintiff. Because this is an aggregate settlement of all of the clients' claims, the attorney must obtain the clients' informed consent after disclosing "the existence and nature of all the claims" and "the participation of each person in the settlement." Model Rule 1.8(g). The necessary disclosure must include information about "all the material terms of the settlement, including what the other clients will receive or pay if the settlement or plea offer is accepted." Model Rule 1.8(g), Comment [13]. Even if the settlement could be jeopardized, Model Rule 1.2(a) protects each client's right to have the final say in deciding whether to accept or reject a settlement. Ultimately, it is not the lawyer's place to decide what is the best outcome for the clients.

An attorney represented a client who was injured when the television antenna he was attempting to erect in his yard came in contact with a power line. As part of its defense, the manufacturer of the antenna claimed that the antenna came with a warning label advising against erecting the antenna near power lines. The client told the attorney that he had not seen a warning label. The client's wife told the attorney that she had kept the antenna and the box it came in and that she saw no warning label anywhere. When called by the attorney as witnesses, both the client and his wife testified that they had never seen a warning label. After their testimony, but while the trial was still in progress, the attorney learned from the wife's sister that there indeed had been a warning label on the box, but that the wife had removed and destroyed it. When the attorney confronted the wife with her sister's statement, the wife admitted destroying the label but insisted that her husband knew nothing about it. The attorney continued the trial, but made no reference to the absence of a warning label in his summation to the jury. Instead, the attorney argued that the warning label, even if seen, was insufficient to advise his client of the serious consequences that would ensue if the warning was not heeded. The jury found in favor of the manufacturer. Is the attorney subject to discipline? a. Yes, because the attorney failed to take reasonable remedial action after he realized that the wife had given perjured testimony. b. No, because the jury apparently disbelieved the wife's testimony. c. Yes, because the attorney called the wife as a witness and she gave perjured testimony. d. No, because the attorney did not rely on the wife's testimony once he discovered the perjury.

a. Yes, because the attorney failed to take reasonable remedial action after he realized that the wife had given perjured testimony. This is the correct answer. "If a lawyer, the lawyer's client, or a witness called by the lawyer, has offered material evidence and the lawyer comes to know of its falsity, the lawyer shall take reasonable remedial measures, including, if necessary, disclosure to the tribunal." Model Rule 3.3. Simply not referring to the absence of the warning label in his summation is not a valid remedial measure.

An attorney agreed to represent a client who was being prosecuted for driving while intoxicated in a jurisdiction in which there is an increased penalty for a second offense. The client told the attorney that his driver's license had been obtained under an assumed name because his prior license had been suspended for driving while under the influence of alcohol. Obtaining a license in this way was a felony. The client asked the attorney not to disclose his true name during the course of the representation and told the attorney that, if called as a witness, he would give his assumed name. The attorney informed the client that, in order to properly defend the case, the attorney must call the client as a witness. The attorney called the client as a witness and, in response to the attorney's question "What is your name?" the client gave his assumed name. Is the attorney subject to discipline? a. Yes, because the attorney knowingly used false testimony. b. Yes, because the client committed a felony when he obtained the driver's license under an assumed name. c. No, because the client's true name was not an issue in the proceeding. d. No, because the attorney's knowledge of the client's true name was obtained during the course of representation

a. Yes, because the attorney knowingly used false testimony. A lawyer may not counsel or assist a client in conduct that is criminal or fraudulent. Model Rule 1.2(d). Similarly, a lawyer has a duty under Model Rule 3.3(a)(3) not to use false evidence. When the attorney asked the client for his name, he knew that the response would be a falsehood that would mislead the court in its judgment of the client's offense.

An attorney is representing a client on a charge of armed robbery. The client claims that the prosecution witness is mistaken in her identification. The client has produced a witness who will testify that the client was in another city 500 miles away when the robbery occurred. The attorney knows that the witness is lying, but the client insists that the witness be called on the client's behalf. The attorney is not planning on relying on the alibi defense in her argument before the jury. Is the attorney subject to discipline if she calls the witness? a. Yes, because the attorney knows the witness will be testifying falsely. b. Yes, because before calling the witness, the attorney did not inform the court of her belief. c. No, because the attorney will not rely on the alibi defense in her argument before the jury. d. No, because the client has insisted that the witness be called as a witness on the client's behalf.

a. Yes, because the attorney knows the witness will be testifying falsely. "A lawyer shall not knowingly ... offer evidence that the client knows to be false." MR 3.3(a)(3). Because the attorney knows the witness is lying, she cannot offer the witnesse's testimony in court.

An inventor retains an attorney to assist with licensing his patented invention. The inventor accurately describes the invention, an electric vehicle battery, to the attorney as capable of powering an electric vehicle further and faster - but not longer before recharge - than other comparable batteries. On the inventor's behalf, the attorney contacts another lawyer, who represents a manufacturer, about licensing and producing the invention. The attorney describes the invention as capable of powering an electric vehicle the "furthest, fastest, and longest before recharge of any battery ever produced." Will the attorney be subject to discipline? a. Yes, because the attorney made a misrepresentation of material fact to the lawyer. b. Yes, because the attorney overstated the invention's capabilities. c. No, because the attorney's description of the invention constituted "puffing" for sales purposes, rather than a false statement. d. No, because the attorney made his statement to the lawyer instead of the manufacturer.

a. Yes, because the attorney made a misrepresentation of material fact to the lawyer. This is the correct answer because the attorney improperly made a false statement of material fact to a third party, which MRPC 4.1(a) prohibits in the course of representation. This false statement is that the battery operated "the longest before recharge of any battery ever produced." The attorney said this despite the inventor's prior statement to the contrary. This third factor about the invention's novelty could have been decisive in either the lawyer's or the manufacturer's decision-making process regarding whether to agree to license the invention.

An attorney represented a landlord in a variety of matters over several years. The attorney's engagement letter did not specifically state that he would provide anything other than legal advice. An elderly widow living on public assistance filed suit against the landlord alleging that the landlord withheld without justification the security deposit on a rental unit that the widow vacated three years ago. She brought the action for herself, without counsel, in small claims court. The attorney investigated the claim and learned that it was legally barred by the applicable statute of limitations, although the widow's underlying claim was meritorious. The attorney told the landlord of the legal defense, but emphasized that the widow's claim was just and that, in all fairness, the security deposit should be returned to her. The attorney told the landlord: "I strongly recommend that you pay the widow the full amount with interest. It is against your long-term business interests to be known in the community as a landlord who routinely withholds security deposits even though the tenant leaves the apartment in good condition. Paying the claim now will prevent future headaches for you." Ultimately, the landlord paid the widow's claim. Was the attorney's conduct proper? a. Yes, because the attorney may refer to both legal and nonlegal considerations in advising a client. b. No, because in advising the landlord to pay the full claim, the attorney failed to represent zealously the landlord's legal interests. c. No, because the attorney's engagement letter did not inform the landlord that the attorney's advice on the matter would include both legal and nonlegal considerations. d. Yes, because the landlord did not object to the attorney's advice and paid the widow's claim.

a. Yes, because the attorney may refer to both legal and nonlegal considerations in advising a client. This is the correct answer. When a lawyer gives a client advice, he or she may rely on considerations other than those involved in giving purely technical legal advice. Model Rule 2.1. Consequently, it is proper for a lawyer to rely on moral, economic, and social factors in doing so, which is what the attorney did here.

A law firm agreed to represent a client in various business matters. The written retainer agreement called for the client to pay hourly rates of $180 per hour for a partner's time and $110 per hour for an associate' s time. The representation proceeded and the firm submitted monthly bills, which the client paid promptly, After two years, the firm decided to increase their hourly rates by $10. The firm thereafter billed the client at their new rates, but did not specifically inform the client of the increase. The client continued to pay monthly bills promptly. Are the firm's partners subject to discipline? a. Yes, because the client did not consent to the increase. b. No, because the $10 hourly increase is reasonable. c. Yes, because the entire original fee agreement was required to be in writing d. No, because the client agreed in writing to pay the firm's hourly rate.

a. Yes, because the client did not consent to the increase. Under MR1.5(b), any changes in the basis or rate of the fee or expenses shall be communicated to the client. Here, there was no indication that the client had any notice of the change in fee. The necessity of informing a client of a fee change and having the client consent to the increase is particularly apparent when an attorney represents an organization, as payments may be automated or done by some other person or department who may have no idea what the agreed-upon arrangement is.

An attorney's standard retainer contract in divorce cases provides for the payment of a fee of one-third of the amount of alimony or property settlement secured by the attorney. Is the attorney subject to discipline? a. Yes, because the fee is contingent. b. No, because this helps clients who may not otherwise be able to afford an attorney. c. Yes, because a lawyer may not acquire a proprietary interest in a cause of action. d. No, because clients consent to the fee arrangement by signing the retainer contract.

a. Yes, because the fee is contingent. "A lawyer shall not enter into an arrangement for, charge, or collect: (1) any fee in a domestic relations matter, the payment or amount of which is contingent upon securing of a divorce or upon the amount of alimony or support, or property settlement in lieu thereof." MR 1.5(d)(1). Here, the attorney's standard retainer contract sets up a prohibited contingent fee arrangement.

Jaiden, an assistant General Counsel of Plenum, Inc., a pharmaceutical manufacturer, was asked to investigate whether the research division of Plenum was concealing reports of adverse reactions to the company's best-selling product, Somalox, an anti-depressant. Two users of some logs had contacted the general counsel's office, reporting that they had submitted adverse reaction reports to the research division, as directed on the package insert, but that they had received no response from the company. One of these two users alleged that he had suffered hallucinations and suicidal ideation after taking Somalox, and that he had described these reactions in his earlier report. The other user reported that she was hospitalized for depression after having taken Somalox for several weeks. She had reported this to the research division. The non-response is worrisome. The company policy is to acknowledge receipt of any adverse reaction reports and to assure users that the company will report adverse reactions to the Food and Drug Administration as required by law. The research division would route any adverse reaction reports through the general counsel's office to the FDA, but none have been received or sent for Somalox. Jaiden schedules a meeting with Bertha, the chief of the research division. Jaden shows her the correspondence sent to the general counsel's office by the two users and asked her to show him these and any other adverse reaction reports that the company has received. Bertha flushes and stammers in response to Jaden's query. "I am afraid that I have let a few of these slip through the cracks. Since you are the company's lawyer, I can speak to you in confidence, right? Am I in trouble here?" In answering Bertha's questions, which of the following statements would Jaiden be best advised to make? a. "Our conversation is protected by attorney-client privilege, so you can talk with me in confidence." b. "I represent Plenum, so I cannot give you any legal advice except the advice to get a lawyer." c. "I have to report anything that you tell me to the company's leadership, because I represent the company, but I can assure you that the information won't be shared outside of the company-for example, with law-enforcement agencies." d. "You can speak to me in confidence, because you are an employee of Plenum, and therefore your interests are aligned with those of the company."

b. "I represent Plenum, so I cannot give you any legal advice except the advice to get a lawyer." This provides Jaiden's best response. Comment 10 following Rule 1.13 states: There are times when the organization's interest may be or become adverse to those of one or more of its constituents. In such circumstances the lawyer should advise any constituent, whose interest the lawyer finds adverse to that of the organization of the conflict or potential conflict of interest, that the lawyer cannot represent such constituent, and that such person may wish to obtain independent representation. Care must be taken to assure that the individual understands that, when there is such adversity of interest, the lawyer for the organization cannot provide legal representation for that constituent individual, and that discussions between the lawyer for the organization and the individual may not be privileged. Rule 4.3 offers similar guidance for any lawyer dealing with an unrepresented person.

Last year, attorney Mira represented Sally, the owner of a small pizza restaurant, in a suit against Rome's Own, Inc., a supplier of shredded cheese, based on breach of contract. The parties settled the claim for $15,000. Mira and Sally remained friendly after the matter was settled, and have gone to the movies and to dinner a few times in the last year. Earlier today, Sally's next-door neighbor, Jose, came into Mira's office for a consultation and asked Mira to represent him in a civil suit against Sally. Jose alleges that Sally's pet macaw bit him, causing an infection and permanent scarring. Jose wants to get a court order against Sally requiring that the bird to be euthanized pursuant to a state law. Sally already paid for his medical care. Jose does not want further damages. Must Mira obtain Sally's informed consent before she can agree to represent Jose? a. No, because Jose's interests are not materially adverse to Sally's. b. No, because Jose's lawsuit is not substantially related to the breach of contract lawsuit against Rome's Own, Inc. c. Yes, because here is a significant risk that Mira's representation of Jose will be materially limited by her loyalty to Sally. d. Yes, because Jose's interests are directly adverse to Sally's.

b. No, because Jose's lawsuit is not substantially related to the breach of contract lawsuit against Rome's Own, Inc. This is the correct answer. Because Sally is Mira's former client, Rule 1.9 applies. Under Rule 1.9(a), an attorney must obtain a former client's informed consent only if she plans to represent another person in the "same or a substantially related matter in which that person's interests are materially adverse to the interests of the former client." The question of substantial relationship is whether there is a substantial risk that Mira would normally have learned confidential information in her former representation of Sally that would materially advance Jose's position in the new matter. If Jose were seeking damages, Mira might have knowledge of Sally's financial situation that might be used adversely to Sally. However, since Jose is only seeking an injunction it is unlikely that any prior confidences could be use adversely. Therefore there probably is not a substantial relationship between the prior matter and the current one, so no consent is required.

An attorney has been hired by a client to represent the client in a civil commitment proceeding initiated by the state. The client is now undergoing psychiatric evaluation to determine whether civil commitment should be ordered. The client told the attorney that the client intends to commit suicide as soon as the tests are completed, and the attorney believes that the client will carry out this threat. Suicide and attempted suicide are crimes in the state. Is it proper for the attorney to disclose the client's intentions to the authorities? a. No, because disclosure would aid the state in its civil commitment case against the client. b. Yes, because the information concerns a future crime that is likely to result in the client's imminent death. c. No, because the attorney does not know whether the client has attempted suicide in the past. d. Yes, because the information concerns a future crime and is not protected by the attorney-client evidentiary privilege.

b. Yes, because the information concerns a future crime that is likely to result in the client's imminent death. A lawyer may reveal information learned in confidence if disclosure of that information is necessary to prevent reasonably certain death or substantial bodily harm. Model Rule 1.6(b)(1). Here, if the attorney does not reveal the information, it will likely result in the death of the client. Consequently, it would be proper for the attorney to disclose the client's intentions to the authorities.

Spencer, a businessman, calls Kai, an attorney. He says he is looking for a lawyer so that he can sue his former business partner, Max, with whom he has had a falling out. He asserts that he has proof that Max was stealing money from his company and covering it up by keeping two sets of books. Spencer says he has photos of several pages from both sets of books, which he obtained by sneaking into Max's house, where he found the books in the study and photographed them with his cell phone. Kai makes an appointment to meet with Spencer the following week to discuss possible representation. The day after the phone call, Max calls Kai to ask for representation because he thinks Spencer is going to sue him. Max offers Kai a generous flat fee for the work. Kai would like to represent Max. May Kai represent Max without Spencer's consent? a. No, because an attorney may not accept representation of a person where the attorney's representation has first been requested by a potential adversary of that person. b. No, because Spencer has given Kai information that can be used adversely to Spencer if Kai represent Max. c. Yes, because there is no conflict of interest, since Kai has not yet agreed to represent Spencer. d. Yes, because Spencer's actions were not lawful.

b. No, because Spencer has given Kai information that can be used adversely to Spencer if Kai represent Max. Spencer is a prospective client under rule 1.18 because he has consulted with Kai about the possibility of becoming Kai's client. Under rule 1.18(c), Kai may not represent Max because he has received information from Spencer about his unlawful break-in. That information could be used adversely to Spencer if Spencer sues Max.

An assistant district attorney was in charge of the presentation before a grand jury of evidence that led to an indictment charging 32 defendants with conspiracy to sell controlled drugs. Shortly after the grand jury returned the indictments, the attorney resigned as assistant district attorney and became an associate in the law office of an attorney for one of the indicted codefendants. The former district attorney did not reveal any confidence or secret learned while he was an assistant district attorney, but he also did not expressly state he would not participate in the case. Is it proper for that attorney to continue to represent the codefendant? a. No, because the former assistant district attorney did not agree to not participate in the representation of the codefendant b. No, because the former assistant district attorney had substantial responsibility for the indictment of the codefendant. c. Yes, because the former assistant district attorney did not reveal to the attorney any confidence or secret learned while an assistant district attorney. d. Yes, because a public prosecutor must make timely disclosure to the defense attorney of any exculpatory evidence.

b. No, because the former assistant district attorney had substantial responsibility for the indictment of the codefendant. This is the correct answer. These facts invoke the concepts of conflicts of interest and imputed disqualification. Model Rule 1.9 deals with the conflict faced by a lawyer who has represented one client, terminates the representation, and then is asked to represent a new client with interests adverse to those of the lawyer's former client. If you simplify these facts, it's clear that the former assistant district attorney could not himself represent the codefendant after he participated in the investigation of the codefendant because the interests of the codefendant are adverse to the former assistant attorney's interests. Model Rule 1.9(a). But the question asks about the attorney's representation of the codefendant, not the former assistant district attorney's. Model Rule 1.10(a) extends the former assistant district attorney's disqualification to the attorney under the imputed disqualification rule: "While lawyers are associated in a firm, none of them shall knowingly represent a client when any one of them practicing alone would be prohibited from doing so by Model Rules 1.7 or 1.9."

A husband and wife are experiencing serious marital difficulties. The wife retains an attorney, who files a divorce action on behalf of the wife. Another lawyer agrees to represent the husband in the action. The extent of the parties' irreconcilable differences has cause them to become very adversarial. Moreover, the wife engaged in an extramarital affair. Feeling particularly vindictive, the husband tells his lawyer that he wants him to take an aggressive approach to litigation. The husband says: "My wife does not deserve any favors or concessions from us. We need to win this case even if it's on procedural grounds." The husband's lawyer serves the wife's attorney with a notice of taking the wife's deposition. The wife's attorney writes the husband's lawyer with a request that he reschedule the deposition because it is set for the same date as the wife's surgery for a life-threatening condition. The husband objects to rescheduling the deposition. Will the husband's lawyer be subject to discipline if he agrees to reschedule the deposition? a. Yes, because the lawyer lacked the husband's approval to reschedule the deposition. b. No, because the husband's rights would not be prejudiced by rescheduling the deposition. c. Yes, because the lawyer would not be following the husband's directive. d. No, because the husband has the right to determine the means and objectives of legal representation

b. No, because the husband's rights would not be prejudiced by rescheduling the deposition. This is the correct answer for two reasons, despite the husband's directive that the lawyer must take an aggressive approach to litigation. First, the lawyer can exercise reasonable discretion in the conduct of litigation by agreeing to reschedule the deposition. MRPC 1.2 provides that a client determines the objectives of the representation, but the means of accomplishing those objectives is generally within an attorney's discretion. Second, the lawyer will not be subject to discipline because such an agreement would not prejudice the husband's rights.

An attorney represents a wife in a marriage dissolution proceeding that involves bitterly contested issues of property division and child custody. After one day of trial, the husband, through his attorney, made a settlement offer. Because of the husband's intense dislike for the attorney, the proposed settlement requires that the attorney agree not to represent the wife in any subsequent proceeding, brought by either party, to modify or enforce the provisions of the decree. The wife wants to accept the offer, and the attorney believes that the settlement offer made by the husband is better than any award the wife would get if the case went to judgement. Although the attorney thought it was in the wife's best interest to accept the offer, he was concerned that another attorney may not be able to adequately protect the wife's interests in the future. Is it proper for the attorney to agree that he will not represent the wife in any subsequent proceeding? a. Yes, because the attorney believes that it is in the wife's best interests to accept the proposed settlement. b. No, because the proposed settlement would restrict the attorney's right to represent the wife in the future. c. No, because the attorney did not believe that the wife's interests can be adequately protected by another lawyer in the future. d. Yes, because the restriction on the attorney is limited to subsequent proceedings in the same matter.

b. No, because the proposed settlement would restrict the attorney's right to represent the wife in the future. This is the correct answer. Under Model Rule 5.6(b), an attorney shall not participate in offering or making "an arrangement in which a restriction on the lawyer's right to practice is part of the settlement of a client controversy." Consequently, the proposed settlement violates the Rules.

A client has asked an attorney to represent her in obtaining compensation for a tract of land that is being condemned by the state department of transportation to build a new highway. Two years ago, the attorney had been employed by the department and had been assigned to search title on several tracts of land, including the one owned by the client. The attorney remembers a department engineer had drafted a confidential memorandum advising against running a new highway across the client's land because of a potential adverse environmental impact. Because of this information, the attorney believes it is possible to prevent the condemnation of the client's land or to increase the settlement amount. What is the proper action for the attorney to take? a. Represent the client on the issue of damages only and not disclose the information that might prevent the condemnation. b. Refuse to represent the client and not disclose the information about the adverse environmental impact. c. Represent the client and attempt to prevent the condemnation by using the information about the adverse environmental impact. d. Refuse to represent the client but disclose to her the information about the adverse environmental impact.

b. Refuse to represent the client and not disclose the information about the adverse environmental impact. This is the correct answer. A lawyer who was formerly employed by a government agency may not later represent a client "in connection with a matter in which the lawyer participated personally and substantially" as a public employee, unless the government agency consents in writing. Model Rule 1.11(a)(2). If the lawyer is practicing in a firm, this disqualification would cover any other lawyer in the firm unless the firm puts together a screening arrangement and gives written notice to the agency. Model Rule 1.11(b). Here, the attorney participated personally and substantially by conducting a title search on the client's property when he was employed by the government agency. Furthermore, the attorney may not disclose the confidential information he learned while working at the department because a lawyer who was formerly employed by a government agency must not reveal confidential information relating to a former representation. Model Rules 1.11(a)(1) and 1.9(c).

An attorney is defending a corporation against a lawsuit brought in federal court by the plaintiff, a consumer injured by one of the corporation's products. The plaintiff is seeking both compensatory and punitive damages. During discovery, the plaintiff's lawyer served a set of interrogatories on the corporation, including requests for the financial data of the corporation. The president of the corporation directed the attorney to resist providing this information, although the attorney has informed him that, under the rules of discovery, the plaintiff is entitled to the information requested. The president then demanded that the attorney assert that the information is confidential, privileged work product, and a trade secret, but the attorney correctly informed him that it was well settled that such claims would be regarded as frivolous by the court. The president nonetheless directed the attorney to file objections on the bases stated, so that at least the plaintiff will have to incur the expense of compelling discovery. The attorney filed the objections as directed by the president. Which of the following statement is true? a. The attorney is subject to neither discipline nor litigation sanction. b. The attorney is subject to both discipline and litigation sanction. c. The attorney is subject to discipline. d. The attorney is subject to litigation sanction.

b. The attorney is subject to both discipline and litigation sanction. This statement is true. By doing what the president asked, the lawyer has violated Model Rule 3.1, which requires meritorious claims and contentions; Model Rule 3.2, which requires the lawyer to expedite litigation; Model Rule 3.3, which requires candor toward the tribunal; and Model Rule 3.4, which requires fairness to the opposing party and counsel. Consequently, the lawyer is subject to discipline under Model Rule 8.4. Additionally, Federal Rules of Civil Procedure Rule 11 provides litigation sanctions for an attorney who files a pleading or other motion for "any improper purpose, such as to harass, cause unnecessary delay, or needlessly increase the cost of litigation."

An attorney is a senior partner at a law firm in which there are 50 lawyers. The firm pays each of its lawyers a fixed annual salary. In addition, at year's end, each lawyer receives a bonus from the profits of the firm in proportion to the annual salary of each and its relation to the total of the fixed annual salaries of all lawyers. The attorney plans to introduce a new management plan under which the firm's nonlawyer office administrator would have general charge of all business matters but would not participate in any decisions involving legal judgment. The administrator would be paid a fixed annual salary and would be included as a participant in the firm's bonus plan on the same basis as the lawyers in the firm. This would usually yield a bonus of approximately one-fourth to one-third of the administrator's total annual compensation. The amount paid to the administrator will not exceed the compensation commonly paid to law office administrators within the local legal community. Is it proper for the attorney to institute such a plan? a. Yes, because the amount paid to the administrator does not exceed the compensation commonly paid to law office administrators within the local legal community. b. Yes, because an employee of the firm may be compensated based on the profits of the firm. c. No, because the administrator's bonus is computed on the same basis as those of the lawyers in the firm. d. No, because the administrator's compensation is derived from the legal fees of the firm's lawyers.

b. Yes, because an employee of the firm may be compensated based on the profits of the firm. This is the correct answer. This plan for part of the administrator's compensation to be based upon the net profits of the firm is expressly permitted under Model Rule 5.4(a) as long as the nonlawyer is an employee of the firm. Importantly, the other aspects of the arrangement do not give the administrator an inappropriate ownership interest in the firm, nor in any right to control the legal work done in the law firm, thus avoiding any violation of the Rule.

A corporation has applied to a bank for a $900,000 loan to be secured by a lien on the corporation's inventory. The inventory, consisting of small items, constantly turns over. The security documents are complex and if improperly drawn they could result in an invalid lien. The bank has approved the loan on the condition that the corporation and the bank jointly retain an attorney to prepare the necessary security instruments and that the corporation pay the attorney's fees. Both the corporation and the bank gave informed consent in writing to the attorney's representation of both parties. This arrangement is customary in the city in which the attorney's law office and the bank are located. It is obvious to the attorney that he can adequately represent the interests of both the corporation and the bank. Is it proper for the attorney to prepare the security documents under these circumstances? a. No, because the attorney's fees are being paid by the corporation, not the bank. b. Yes, because both the bank and the corporation have given their informed consent to the arrangement. c. No, because the corporation and the bank have differing interests d. Yes, because the arrangement is customary in the community

b. Yes, because both the bank and the corporation have given their informed consent to the arrangement. Under Model Rule 1.7(a), a lawyer shall not represent a client if the representation of one client will be directly adverse to the representation of another client. Here, the attorney has a conflict of interest arising from the representation of two clients in the same transaction - the bank, which is the lender, and the corporation, which is the borrower. However, Model Rule 1.7(b) allows the attorney to represent both clients if he reasonably believes that he will be able to provide competent and diligent representation to each client, the representation is not prohibited by law, the representation does not involve the assertion of a claim by one client against the other, and each client gives informed consent in writing. Here, because the proposed representation meets all the requirements of Model Rule 1.7(b), the joint representation is proper.

Donald was suspected of having robbed a liquor store. Shortly after he was arrested, he demanded to see his lawyer, Craig. The police did not attempt to question Donald, and Craig arranged for him to be released on bail at his arraignment, after Donald was read the charges filed against him and given a court date. Craig filed papers informing the court and the prosecutor's office that he would be representing Donald in the matter. Brenda, a prosecutor, was assigned responsibility for Donald's case. A state statute authorizes prosecutors to investigate criminal cases by questioning witnesses or authorizing police officials to do so up to the point at which charges are filed. Brenda visited Donald's home a week after he was released, identified herself as the prosecutor in the case, and asked if she might ask him some questions. Donald said he would answer her questions because he was innocent of the charges and therefore had nothing to hide. Brenda did not make any false or misleading statements to Donald. On the basis of information from his answers, Brenda was able to contact and interview other witnesses. She did not offer into evidence any information that she received from Donald. Is Brenda subject to discipline? a. No, because prosecutors are authorized by law to interview defendants. b. Yes, because she did not first obtain Craig's consent to talk to Donald. c. Yes, because she did not first notify Donald of his right to have counsel present at the interview. d. No, because she did not make any misleading statements to Donald.

b. Yes, because she did not first obtain Craig's consent to talk to Donald. This is correct. Rule 4.2 requires consent of opposing counsel before a lawyer, including a prosecutor, talks to an opposing party known to be represented by counsel in the matter.

An attorney is widely regarded as an exceptionally competent practitioner in the field of criminal law. A client of the attorney became the subject of a grand jury investigation in a matter that could result in a felony indictment. The client lacked sufficient funds to pay for the attorney's services beyond the grand jury stage. He asked the attorney to provide limited representation for a flat fee. Under the arrangement he proposed, the attorney would advise the client concerning the grand jury investigation, but the representation would end when an indictment was returned or the grand jury decided not to indict. The attorney fully advised the client of the practical and legal aspects of the client's proposal. Is it proper for the attorney to accept this limited representation? a.Yes, because the client and not the attorney suggested the arrangement b. Yes, because the attorney and the client may agree to limit the scope of the representation so long as the limitation is reasonable under the circumstances. c. No, because the attorney should not limit the scope of the representation based on the client's ability to pay. d. No, because the scope of the representation may not be limited in a criminal case.

b. Yes, because the attorney and the client may agree to limit the scope of the representation so long as the limitation is reasonable under the circumstances. As long as the client is fully informed, a lawyer and client may agree to limit the scope of the lawyer's representation so long as the limitation is reasonable under the circumstances. MR 1.2(c). Although clients usually retain a lawyer to take care of all aspects of a particular matter, if the client affirmatively consents after consultation, the client can limit the scope of the representation to only one part of the claim. Consequently, it is clearly permissible under the Rules for the client and the lawyer to agree to a representation that only covers the grand jury proceeding.

An attorney represented the plaintiff in an automobile accident case. Two weeks before the date set for trial, the attorney discovered that there was an eyewitness to the accident. The attorney interviewed the witness. Her version of the accident was contrary to that of the plaintiff and, if believed by the trier of fact, would establish that the plaintiff was at fault. The witness told the attorney that she had not been interviewed by defense counsel. The witness also told the attorney that she was uncomfortable with testifying and that she had been thinking about taking a vacation to Europe the following week. The attorney told the witness that, since no one had subpoenaed her yet, she had no obligation to appear. He told her that trials were very difficult for witnesses and suggested that she take the vacation so that she would be unavailable to testify. It the attorney subject to discipline? a. Yes, because the attorney did not subpoena the witness knowing she was an eyewitness. b. Yes, because the attorney asked the witness to leave the jurisdiction. c. No, because the witness had not been subpoenaed by the defense. d. No, because the attorney did not offer the witness any inducement not to appear at the trial.

b. Yes, because the attorney asked the witness to leave the jurisdiction. This is the correct answer. A lawyer is prohibited form asking a person other than the client to refrain from voluntarily giving relevant information to another party unless that person is a relative, employee, or agent of the client and the lawyer reasonably believes that the person's interests will not be adversely affected by refraining to give that information. Model Rule 3.4(f). In that case, the attorney may instruct the person to refrain from giving information unless subpoenaed. Here, the eyewitness did not fall under any of the exceptions to the Rule.

An attorney practices law in the same community as a lawyer who is running as a state judge. The attorney has frequently observed the judicial candidate's courtroom demeanor in litigated cases. Based on those experiences, the attorney believes that the judicial candidate does not have a proper judicial temperament. A local news reporter asked the attorney how he would rate the candidate, and the attorney responded in good faith that he believed the candidate was unsuited for the bench and lacked the proper judicial temperament for a judge. A local newspaper with a wide circulation quoted the attorney's remarks. Were the attorney's remarks proper? a. Yes, because the attorney was not seeking judicial office. b. Yes, because the attorney believed the candidate was unsuited for the bench. c. No, because the remarks serve to bring the judiciary into disrepute. d. No, because a lawyer should not publicly comment on candidates for judicial office.

b. Yes, because the attorney believed the candidate was unsuited for the bench. This is the correct answer. "A lawyer shall not make a statement that the lawyer knows to be false or with reckless disregard as to its truth or falsity concerning the qualifications or integrity of a judge, adjudicatory officer, or public legal officer, or of a candidate for election or appointment to judicial or legal office." Model Rule 8.2(a). However, lawyers' assessments are necessarily relied on in determining the fitness of persons being considered for election or appointment to judicial offices. Model Rule 8.2, Comment [1]. Consequently, the attorney's expression of good faith views about a judicial candidate's qualifications was proper, because "[e]xpressing honest and candid opinions on such matters contributes to improving the administration of justice." Model Rule 8.2, Comment [1]. The Rule targets false statements, not honest opinions.

An attorney represents a corporation that is a defendant in a product liability case. An engineer who is a corporation employee and nearing retirement was likely to be a key witness in the case, as she had been in charge of all of the corporation's product safety testing during the relevant period. The engineer had been very critical of the corporation's safety testing procedures during that period and had repeatedly complained that the product at issue had not been adequately tested. The engineer's views were reduced to writing and were well known to many employees of the corporation. Because of the early stage of the case, however, plaintiff's counsel was not yet aware of the engineer's existence or her views. Aware of the engineer's views, the attorney approached the corporation's officials and recommended that it offer the engineer a special package of severance benefits if she would retire immediately and move to the Bahamas. The attorney believed that if the engineer accepted this offer, she would be beyond the subpoena power of the court in which the suit against the corporation was pending. The corporation adopted the attorney's recommendation and made the offer. The engineer accepted it. The attorney did not disclose the engineer's identity to plaintiff's counsel. It the attorney subject to discipline? a. No, because the engineer's views were reduced to writing and are well known to many other employees of the corporation. b. Yes, because the attorney caused the engineer to leave the jurisdiction of the court for the purpose of making her unavailable as a witness. c. No, because there was no pending request for the engineer's testimony at the time the retirement offer was made to the engineer. d. Yes, because opposing counsel had not yet had a reasonable opportunity to learn of the engineer's views.

b. Yes, because the attorney caused the engineer to leave the jurisdiction of the court for the purpose of making her unavailable as a witness. This is the correct answer. "A lawyer shall not: (a) unlawfully obstruct another party's access to evidence." Model Rule 3.4(a). "Fair competition in the adversary system is secured by prohibitions against destruction or concealment of evidence, improperly influencing witnesses, obstructive tactics in discovery procedure, and the like." Model Rule 3.4, Comment [1]. Here, the attorney's actions in putting the engineer out of reach of the court are clearly improper.

An attorney is representing the plaintiff in a paternity suit against a defendant. Both the plaintiff and the defendant are well known public figures, and the suit has attracted much publicity. The attorney has been billing the plaintiff at an agreed hourly fee for his services. Recently, the plaintive told the attorney, "I'm going broke paying you. Why don't you let me assign you all media rights to books, movies, or television programs based on my suit as full payment for all services you will render me between now and the conclusion of the suit?" The attorney replied, "I'll consider it, but first you should seek independent advice about whether such an arrangement is in your own best interest. Why don't you do so and call me next week?" The plaintiff received independent advices and entered into the agreement soon thereafter. Is the attorney subject to discipline if he agrees to the plaintiff's offer? a. Yes, because the amount received by the attorney would be contingent on the receipts from the sale of media rights. b. Yes, because the attorney has not concluded the representation of the plaintiff. c. No, because the paternity suit is a civil and not a criminal matter. d. No, because the plaintiff received independent advice before entering into the agreement.

b. Yes, because the attorney has not concluded the representation of the plaintiff.

A trust company entered into the following arrangement with an attorney who was newly admitted to the bar: The trust company would provide the attorney with free office space in the building in which the trust company had its offices. If a customer of the trust company contacted the trust company about a trust or will, an officer of the trust company, who would not be a lawyer, would advise the customer and help the customer work out the details of the trust or will. The customer would be informed that the necessary documents would be prepared by the trust company's staff. The completed documents would be submitted by an officer of the trust company to the customer for execution. The attorney, in accordance with a memorandum from the trust company's trust officer detailing the plan, would prepare the necessary documents. The attorney would never meet with the customer and would not charge the customer for these services. The attorney would be free to engage in private practice, subject only to the limitation that the attorney could not accept employment adverse to the trust company. Is the attorney subject to discipline for entering into the arrangement with the trust company? a. No, because the attorney is not giving advice to the trust company's customers. b. Yes, because the attorney is aiding the trust company in the practice of law. d. Yes, because the attorney is restricting his right to practice.

b. Yes, because the attorney is aiding the trust company in the practice of law. This is the correct answer. Because the attorney is newly admitted, he may not realize that he's become party to a scheme that enables the trust company to engage in the unauthorized practice of law. Model Rule 5.5(a) says quite simply, "A lawyer shall not practice law in a jurisdiction in violation of the regulation of the legal profession in that jurisdiction, or assist another in doing so." The attorney is enabling the trust company to discuss and "work out" the details of trusts and wills, functions that clearly fall within the parameters of law practice. The attorney is also aiding the trust company and its officer to misrepresent to the customer the way in which the documents are being prepared. The arrangement between the attorney and the trust company is in the nature of a partnership, and Model Rule 5.4(b) prevents a lawyer from forming a partnership with a nonlawyer if the function of the partnership is the practice of law. Also, even if the attorney is not sharing in the trust company's fees directly, he is in effect doing so by getting free office space. Importantly, Model Rule 5.4(a) prohibits fee splitting with nonlawyers. Ultimately, the attorney is subject to discipline for his actions. c. No, because the attorney is not charging the customer for his services.

An attorney was retained by a passenger on a bus why was injured in a collision between the bus and a truck. Although he was busy, his time was not completely occupied with work for other clients. The passenger paid the attorney a retainer of $1,000, and agreed further that the attorney should have a fee of 25 percent of any recovery before filing suit, 30 percent of any recovery after suit was filed but before judgment, and 35 percent of any recovery after trial and judgment. The attorney promptly called the lawyer for the bus company and told him she was representing the passenger and would like to talk about a settlement. The attorney made an appointment to talk to the lawyer for the bus company, but did not keep the appointment. The attorney continued to put off talking to the lawyer for the bus company. Meanwhile, the passenger became concerned because she had heard nothing from her attorney. The passenger called the attorney's office, but was told the attorney was not in and would not call back. The passenger was told not to worry because the attorney would look after her interests. After 10 months had passed, the passenger went to another attorney for advice. The other attorney advised the passenger that the statute of limitations would run in one week and, with the passenger's consent, immediately filed suit for the passenger. Upon the passenger's demand, the attorney refunded the $1,000 the passenger had paid. Is the attorney subject to discipline? a. No, because the passenger's suit was filed before the statute of limitations ran. b. Yes, because the attorney neglected the representation of the passenger. c. No, because the attorney returned the $1,000 retainer to the passenger. d. Yes, because the attorney's time was not completely occupied with work for other clients.

b. Yes, because the attorney neglected the representation of the passenger. MR 1.3 states: "a lawyer shall act with reasonable diligence and promptness in representing a client." Comment [2] states: "a lawyer's workload must be controlled so that each matter can be handled competently." Comment [3] states: "Perhaps no professional shortcoming is more widely were resented than procrastination." The facts here show the typical way a lawyer can end up subject to discipline: 1. Make appointments and ignore them, 2. Ignore the clients phone calls and requests for information progress reports, 3. Fail to keep the client advised, 4. Work on other matters to the exclusion or neglect of the client, and 5. Ignore the passage of time and the inevitable running of the statute of limitations. The key word in this answer is "neglect." Here, the attorney has been guilty of professional neglect. After misuse of client funds, this is the most common basis for lawyer discipline.

An attorney represents a plaintiff in a civil action that was filed a year ago and is about to be set for trial. The plaintiff informed the attorney that he could be available at any time during the months of October, November, and December. In discussing possible trial dates with opposing counsel and the court clerk, the attorney was advised that a trial date on October 5 was available and that the next available trial date would be December 10. Without first consulting the plaintiff, the attorney requested the December 10 trial date because she was representing a defendant in a felony criminal trial that was set for October 20 and she wanted as much time as possible to prepare for that trial. She was not court-appointed counsel in the criminal case. The later trial date will not prejudice the client. Was it proper for the attorney to agree to the December trial date without obtaining the plaintiff's consent? a. No, because the attorney was not court-appointed counsel in the criminal case. b. Yes, because the plaintiff will not be prejudiced by the delay. c. Yes, because a criminal trial takes precedence over a civil trial. d. No, because the attorney should manage her calendar so that her cases can be tried promptly.

b. Yes, because the plaintiff will not be prejudiced by the delay. A lawyer should act with reasonable diligence and promptness when representing a client. MR 1.3. Here, since the civil action was filed a year ago, the client gave his availability through December, the "delay" is only a little over two months, and the client will not be prejudiced by the delay, it is absolutely reasonable for the attorney to take the December 10 trial date.

A district attorney handles the prosecution of an accused for stealing a car from a dealership's inventory lot. The accused refutes the charges on the basis that she was at her boyfriend's residence when the alleged offense occurred. An employee, who works for the dealership, provides trial testimony. The employee testifies that he recognized the accused's picture in a photographic array, which a police officer showed to him. The district attorney presents a recording of the employee's 911 call reporting the event and describing the accused and the car. The judge renders a judgment of conviction and sentences the accused for the offense. After the trial, the accused's lawyer discovers additional evidence that the employee initially picked another person's photograph from the photographic array before the police officer suggested the accused's picture to him. The accused's lawyer learns that the district attorney knew of that evidence, but failed to reveal it to the lawyer. The lawyer's pretrial discovery request did not seek such evidence. Will the district attorney be subject to discipline? a. No, because the lawyer failed to make a pretrial discovery request for that type of information. b. Yes, because the subsequently discovered evidence tended to negate the accused's culpability for the offense. c. No, because the district attorney did not have to initially disclose the additional evidence that was later discovered. d. Yes, because the judge could have identified the accused based on the 911 call recording.

b. Yes, because the subsequently discovered evidence tended to negate the accused's culpability for the offense. This is the correct answer. MRPC 3.8 requires that the district attorney, a prosecutor, timely provide the accused's lawyer with all "exculpatory" evidence that the district attorney knows either tends to either negate guilt or mitigate the offense. Here, the district attorney failed to do that. The district attorney had a duty to initially disclose the additional evidence that was later discovered.

An elected prosecutor plans to run for reelection in six months. Last year, two teenage girls were kidnapped from a shopping center and sexually assaulted. The community was in an uproar about the crime and put pressure on the prosecutor to indict and convict the assailant. Four months ago, a suspect was arrested and charged with the crimes. The trial is scheduled to begin next week. The prosecutor met with the police chief last week to review the evidence in the case. At that time, the prosecutor first learned that, before they were interviewed by the detective in charge of sexual assault crimes, the two victims had been tape-recorded discussing the case between themselves in an interview room. Reviewing the tape, the prosecutor realized that the girls' descriptions of the assailant differed significantly in terms of height, weight, and hair color. When officially interviewed, however, their descriptions matched almost perfectly. The suspect's appointed counsel was busy handling a large caseload of indigent defendants and neglected to seek access to the prosecution's investigative file. The prosecutor was virtually certain that suspect's counsel was unaware of the tape recording. Given the other evidence in the case, the prosecutor reasonably believed that the girls accurately identified the suspect as their assailant. The prosecutor did not reveal the existence of the tape to defense counsel. Is the prosecutor subject to discipline? a. No, because under the adversary system of criminal justice, it is expected that each party will marshal the evidence best supporting its own position. b. Yes, because the tape raises a legitimate question about the victims' eyewitness identification of the suspect as the assailant. c. No, because the prosecutor reasonably believed that the girls accurately identified the suspect as their assailant. d. No, because the suspect's counsel did not submit a request for all mitigating or exculpatory evidence before the start of trial.

b. Yes, because the tape raises a legitimate question about the victims' eyewitness identification of the suspect as the assailant. This is the correct answer. With regard to civil cases, the adversary system contemplates that the evidence in a case is to be marshaled competitively by the parties. Model Rule 3.4, Comment [1]. Likewise, a lawyer's duty not to conceal evidence in a civil case does not carry with it a duty to report or volunteer all relevant information. Prosecutors, however, have special responsibilities. Model Rule 3.8. "A prosecutor has the responsibility of a minister of justice and not simply that of an advocate" (Model Rule 3.8, Comment [1]) and as such, must "make timely disclosure to the defense of all evidence or information known to the prosecutor that tends to negate the guilt of the accused ...." Model Rule 3.8(d). The fact that the girls' descriptions varied significantly at one point, but later matched almost perfectly, raises a definite question as to the accuracy of the identification and "tends to negate the guilt" of the suspect. Here, the prosecutor has a duty to reveal the existence of the tape to the suspect's counsel.

Carlos, a sole practitioner, represents Sharky Products, Inc., a property owner that disputes an adjoining owner's right of access to an alleyway between their buildings. Carlos sent a letter to the other property owner, Berry's Beauty Supply Co, to explain the problem. He received a reply from Berry's lawyer, Eleanor, requesting that Carlos withdraw from representation of Sharky. Eleanor claimed that Carlos was prohibited from handling this matter because five years ago, Carlos represented Matthew, the president and sole owner of Berry's, in a claim against the manufacturer of a lawn care product that he used at his home. The matter took three hours of Carlos' time and Matthew was satisfied with the resulting settlement. Carlos has had no contact with Matthew since then. May Carlos continue to represent Sharky Products, Inc.? a. No, because Matthew is the president and sole owner of Berry's. b. Yes, because there is no substantial relationship between the two matters. c. Yes, because he only worked on Matthew's case for three hours. d. No, because his representation of Sharky is materially adverse to his former client.

b. Yes, because there is no substantial relationship between the two matters. This is the correct answer. The prior representation was of Matthew as an individual in a matter that would not have involved "a substantial risk that confidential factual information as would normally have been obtained in the prior representation would materially advance the client's position in a subsequent matter." See Rule 1.9, comment 3. This is evident because the prior representation was not of Sharky, but of one of its officers, and the representation did not involve financial information about the corporation.

An attorney is defending a client who has been indicted for burglary. During an interview, the client stated to the attorney that before he had consulted the attorney, the client had committed perjury while testifying before the grand jury that indicted him. The attorney is subject to discipline if she: a. continues to represent the client unless he admits his perjury. b. informs the authorities of the perjury. c. continues to represent the client. d. does not inform the authorities of the perjury.

b. informs the authorities of the perjury. This is the correct answer. A lawyer must keep a client's confidences unless the information needs to be revealed to prevent death or substantial bodily harm, crime or fraud, secure legal advice about the lawyer's compliance with the Rules, establish a lawyer's claim or defense, comply with a law or court order, or detect and resolve conflicts. Model Rule 1.6. The information revealed by the client involves none of these potential problems. Importantly, the client committed perjury before the lawyer's representation. Had the perjury occurred during the lawyer's representation, the lawyer would have been required to take remedial measures under Model Rule 3.3(a)(3) But, since the client acted before representation began if would be a violation of the Rules to reveal the client's confidential information.

In the attorney's closing statement to the court in a bench trial, the attorney said, "Your honor, I drive on the street in question every day and I know that a driver cannot see cars backing out of driveways as the one did in this case. I believe that my client was not negligent, and I ask you to so find." The attorney was not trying to deceive the court with his statement. There was no other evidence in the record regarding the facts asserted by the attorney. Was the attorney's closing argument proper? a. No, because there is no other evidence in the record about the facts asserted by the attorney. b. Yes, because the rules of evidence are very liberal when the trial is before a judge without a jury. c. No, because the attorney asserted his personal knowledge of facts in issue. d. Yes, because the attorney was speaking truthfully and not trying to deceive the court.

c. No, because the attorney asserted his personal knowledge of facts in issue. This is the correct answer. During trial, a lawyer may not "assert personal knowledge of facts in issue except when testifying as a witness." Model Rule 3.4(e). Here, by stating his belief regarding the street and driveways, he is clearly violating that Rule.

An attorney who is a sole practitioner limits his practice to personal injury cases. He regularly places advertisements in local newspapers that state that his practice is limited to personal injury cases, including medical malpractice. After seeing one of the attorney's ads, a man approached the attorney for representation in a medical malpractice case. After a 30-minute interview, the attorney told the man that he was too busy to take his case because it appeared quite complicated. He further offered to refer the man to another lawyer who regularly practiced in the field. He reminded the man that he should see another lawyer promptly before the statute of limitations expired and he lost his right to sue. Although the attorney did not charge the man for the interview, the man was upset at wasting 30 minutes of his time. The man did not contact another lawyer until eight months later, when he learned that the statute of limitations on his claim had expired six months after his interview with the attorney. In fact, the man had a meritorious medical malpractice claim. Is the attorney subject to civil liability? a. No, because the attorney offered to refer the man to another medical malpractice lawyer b. Yes, because the attorney falsely advertised his availability for medical malpractice cases. c. No, because the attorney did not violate any duty owed to the man. d. Yes, because the attorney did not advise the man as to the date the statute of limitations would expire.

c. No, because the attorney did not violate any duty owed to the man. Under Model Rule 1.18, a prospective client is a person who discusses with the lawyer the possibility of forming a client lawyer relationship with respect to a certain matter. The Comment to the Rule recognizes that prospective clients should receive some but not all of the protection afforded clients. Here, the attorney talked to the prospective client, declined the representation, gave the man another lawyer's name, and urged him to contact the other lawyer in a timely manner. Under the Rule, the attorney did not owe the prospective client any other duty, except that he must not mislead the prospective client into believing the attorney will represent him, and he must also protect any confidential information revealed by the prospective client. Model Rules 3.3 and 1.6.

An attorney is employed by a client who is a fugitive from justice under indictment for armed robbery. The attorney, after thorough legal research and investigation of the facts furnished by the client, reasonably believes the indictment is fatally defective and should be dismissed as a matter of law. The attorney advised the client of his opinion and urged the client to surrender. The client told the attorney that she would not surrender. The attorney informed the district attorney that he represented the client and that he had counseled her to surrender but that she refused to follow his advice. The attorney has not advised his client on how to avoid arrest and prosecution and does not know where she is hiding. Is the attorney subject to discipline if he continues to represent the client? a. No, because the attorney reasonably believes the indictment is defective. b. Yes, because the client is engaging in continuing illegal conduct. c. No, because the attorney is not counseling the client to avoid arrest and prosecution. d. Yes, because the client refused to accept the attorney's advice and surrender.

c. No, because the attorney is not counseling the client to avoid arrest and prosecution. A lawyer may not knowingly counsel or assist a client in conduct the lawyer knows is criminal or fraudulent. Model Rule 1.2(d). When the client's course of action has already begun and is continuing, the lawyer is required to avoid assisting the client in the criminal act, such as by suggesting how the wrongdoing can be concealed. Here, although the client is engaged in continuing illegal conduct by remaining a fugitive, the attorney is not actually counseling or assisting the client in doing so (in fact, he encouraged the client to surrender). The important point in the Rule is "counsel or assist" in the criminal act. Consequently, the attorney is acting correctly under the Rules.

A county law prohibits stores from selling alcoholic beverages before noon on Sundays. Failure to comply is a misdemeanor punishable by a fine of $150. An attorney was hired by a client who owns several liquor stores. The client asked the attorney whether any storeowners had been prosecuted for violating the law and whether the fine could be imposed for every sale on a Sunday before noon or only for every Sunday on which alcohol was sold before noon. The client also asked what he could do to minimize the risk that he would be detected. The attorney accurately told the client that the fine could only be imposed for each Sunday on which he sold alcoholic beverages before noon, not for each transaction, and that no one had been prosecuted under the law as yet. She also told him that she thought it would be improper to advise him about how to avoid detection. The client thanked the attorney for the information and hung up. Several weeks later, the attorney learned that the client had begun to open his store for business on Sundays at 9 a.m. Is the attorney subject to discipline? a. Yes, because the attorney did not discourage her client from breaking the law. b. No, because the lawyer and the client could have discussed the best way to avoid detection under the criminal law. c. No, because the attorney merely gave the client her honest opinion about the consequences that were likely to result if he violated the law. d. Yes, because the attorney reasonably should have known that the information she gave the client would encourage him to violate the law.

c. No, because the attorney merely gave the client her honest opinion about the consequences that were likely to result if he violated the law. Although a lawyer is not allowed to counsel or assist a client in criminal or fraudulent conduct, a lawyer may help a client determine the validity, scope, meaning, or application of the law to a client's particular situation. Model Rule 1.2(d). Here, the attorney is not helping the client break the law; she is simply delineating the law's boundaries.

The attorney represented a client in a claim involving a breach of the client's employment contract. The case was settled without suit being filed. The proceeds of the settlement were paid directly to the client, who subsequently paid the attorney in full for the attorney's fee and expenses. Thereafter, the attorney did no other work for the client. The client is now being audited by the Internal Revenue Service (IRS). The IRS has asked the attorney for details of the settlement, including the amount claimed for each item of damage and the amounts paid for the items. The attorney reported the request to the client, who told the attorney not to provide the information to the IRS. The attorney believed the disclosure could hurt his client. Is it proper for the attorney to furnish the information to the IRS? a. Yes, because the attorney no longer represents the client. b. No, because the attorney did not believe the disclosure would be beneficial to the client. c. No, because the client told the attorney not to provide the information. d. Yes, because the information does not involved the attorney's work product.

c. No, because the client told the attorney not to provide the information. This is the correct answer. "A lawyer shall not reveal information relating to the representation of a client unless the client give informed consent" or the disclosure falls within one of the exceptions related to preventing crime, defending the attorney, or compliance with law or court order. Model Rule 1.6. The IRS is asking the attorney for information regarding the representation in the employment contract case, and the client has not given his consent for the disclosure. Since there is no indication the disclosure falls within any of the exceptions, it would be improper for the attorney to give the information to the IRS.

An attorney represents a client in commercial litigation that is scheduled to go to trial in two months. Over the past several weeks, the client has disagreed with almost every tactical decision that the attorney has made. Frustrated, the attorney finally said to the client that if she didn't like the way he was handling the lawsuit, perhaps she should get another lawyer. The client was upset at the suggestion and accused the attorney of trying to get out of the case. Reasonably believing that he could no longer work effectively with the client, the attorney sought the client's permission to withdraw from the representation, and the client reluctantly agreed. After giving the client sufficient notice to obtain replacement counsel, the attorney requested the court's permission to withdraw from the litigation, but the court denied the request. May the attorney withdraw from the representation? a. Yes, because the client agreed, and the attorney gave the client sufficient notice to obtain replacement counsel. b. Yes, because the client had made it unreasonably difficult for the attorney to carry out the representation effectively. c. No, because the court denied the attorney's request to withdraw. d. No, because the attorney's withdrawal could cause material prejudice to the client, and the client's agreement was not voluntary.

c. No, because the court denied the attorney's request to withdraw. "When ordered to do so by a tribunal, a lawyer shall continue representation notwithstanding good cause for terminating representation." Model Rule 1.16(c). Here, since the court denied the request, the attorney cannot withdraw. This question is a good example of carefully reading the question.

Five years ago, attorney Barry worked at a large law firm with offices across the country. One of the firm's partners defended Panko, Inc., an appliance manufacturer, against allegations that its toasters had a defect that would cause many of them to catch fire. Barry was not involved in that litigation and did not learn anything about it during his time at the firm. Barry has since opened up a private practice of his own. Ted comes to Barry's new office seeking legal help. Ted tells Barry that his Panko toaster recently overheated and caused a house fire. He would like to sue Panko for damages. May Barry represent him [in] a suit against Panko with Panko's informed consent? a. No, because Ted's interests are materially adverse to those of Panko. b. Yes, because there was not a substantial risk that confidential factual information as would normally have been obtained in the prior representation of Panko would materially advance Ted's position in the subsequent matter. c. Yes, because Barry did not actually acquire confidential information that is material to Ted's lawsuit while he was working at his old law firm. d. No, because the matters are the same or substantially related.

c. Yes, because Barry did not actually acquire confidential information that is material to Ted's lawsuit while he was working at his old law firm. This is the correct answer. Barry moved from one law firm to another. His former firm represented Panko in a similar matter. This situation is therefore governed by Rule 1.9(b), not 1.9(a). Under Rule 1.9(b): "A lawyer shall not knowingly represent a person in the same or a substantially related matter in which a firm with which the lawyer formerly was associated had previously represented a client (1) whose interests are materially adverse to that person; and (2) about whom the lawyer had acquired information protected by Rules 1.6 and 1.9(c) that is material to the matter."

Attorney Pavel is a criminal defense lawyer representing Denny, who is charged with larceny. Denny, a pro bono client, is alleged to have stolen a piece of jewelry from a jewelry box during a house party. In an unrelated matter, Pavel is defending Betty, for a fee, against a criminal charge of arson. Both cases are assigned to be heard by a single judge. During Denny's trial, the prosecutor calls Betty to the stand. She testifies that she was at the party in question and saw Denny go in and out of the room where the alleged theft occurred. She further testifies that she never went into that room. Earlier, Denny had told Pavel that he saw Betty go into the room, and he suggested that she may be the thief. Pavel intends to cross-examine Betty on this point. Is there a conflict of interest? a. Yes, because Betty is a paying client b. No, because the representation does not involve the assertion of a claim by Danny against Betty or vice-versa. c. Yes, because Denny's interests are directly adverse to Betty's. d. No, because Pavel's cross-examination of Betty is impliedly authorized to carry out the representation.

c. Yes, because Denny's interests are directly adverse to Betty's. Pavel must cross-examine Betty in order to represent Denny competently and diligently. This situation presents direct adversity between the two clients. Comment 6 after Rule 1.7 explains "a directly adverse conflict may arise when a lawyer is required to cross-examine a client who appears as a witness in a lawsuit involving another client, as when the testimony will be damaging to the client who is represented in the lawsuit." Betty's testimony may exculpate Denny and inculpate Betty. After the cross-examination, Denny might be acquitted on the charge and Betty indicted. Even if she is not charged with the jewelry theft, the cross-examination could negatively impact her credibility and her potential liability on the arson charge. This is not the same impact as the example in the comment, but, as in the example, the cross-examination could harm a current client.

An attorney is a well-known, highly skilled litigator. The attorney's practice is in an area of law in which the trial proceedings are heard by the court without a jury. In an interview with a prospective client, the attorney said, "I make certain that I give the campaign committee of every candidate for elective judicial office more money than any other lawyer gives, whether it's $500 or $5,000. Judges know who helps them get elected." The prospective client did not retain the attorney. Is the attorney subject to discipline? a. No, because the attorney's statements are true. b. Yes, because the attorney's contributions are made without consideration of candidates' merits. c. Yes, because the attorney implied that the attorney receives favored treatment by judges. d. No, because the prospective client did not retain the attorney.

c. Yes, because the attorney implied that the attorney receives favored treatment by judges. This is the correct answer. Lawyers are officers of the court and are charged with the responsibility not to make statements with reckless disregard for the truth concerning the qualifications or integrity of a judge. Model Rule 8.2. Comment [1] points out that false statements by a lawyer can undermine public confidence in the administration of justice. Here, the attorney has attacked the integrity not only of one judge, but of the entire judiciary. Especially because his work as a skilled litigator brings him before these very judges, who decide both fact and law, the inescapable inference is that he has special influence over them.

A seller was engaged in negotiations to sell his interest in a large tract of land to a buyer who was unrepresented in the transaction. Before the seller went out of town for a few days, he told the buyer to call his attorney if the buyer had any questions about the property. The buyer called the seller's attorney, asked certain questions about the size of the tract, and expressed hesitations concerning the high asking price for the tract. The attorney responded that, based on his experience handling real estate transactions in the neighborhood, the buyer would be getting a lot of property for the price. At the time the attorney spoke to the buyer, the attorney knew that there was a defect in the title and that the buyer's attempt to purchase the seller's interest in the tract would not result in the buyer's acquisition of any interest in the property. Relying on the attorney's assurance, the buyer agreed to make the purchase. Shortly after the sale closed, the buyer discovered that his acquisition was worthless. Is the attorney subject to civil liability to the buyer? a. Yes, because the attorney implied that his opinion regarding the value of the property was a disinterested opinion. b. No, because the buyer was not a client of the attorney. c. Yes, because the attorney knowingly made false representations of fact to the buyer. d. No, because the attorney's statement that the buyer would be getting a lot of property for the money was a statement of opinion regarding the value of the property.

c. Yes, because the attorney knowingly made false representations of fact to the buyer. This is the correct answer. "In the course of representing a client a lawyer shall not knowingly: (a) make a false statement of material fact or law to a third person." (Rule 4.1(a). Here, the attorney knows of the title defect and is basically committing fraud by encouraging the buyer to purchase it.

An attorney is a long-time member of the state legislature and serves on the legislative budget committee that funds the local trial courts in the state. The attorney also maintains a part-time law practice as is permitted in the state. An influential businessperson who regularly makes significant contributions to the attorney's political campaigns asked the attorney to help his uncle, who was involved in a bitter divorce. The attorney called the trial judge sitting on the uncle's case, a personal friend of the attorney. In discussing some upcoming votes of the budget committee with the judge, the attorney mentioned that the uncle was the type of solid citizen and influential person who could help garner support for the budget and thus ensure the economic health of the judicial system. The trial judge ultimately ruled in the uncle's favor. Is the attorney subject to discipline? a. Yes, because the trial judge ruled in the uncle's favor. b. No, because the attorney called the trial judge in her capacity as a legislator and not as the uncle's lawyer. c. Yes, because the attorney used her public position to attempt to influence a tribunal in a pending matter. d. No, because members of the state legislature are permitted by law to engage in part-time legal practice.

c. Yes, because the attorney used her public position to attempt to influence a tribunal in a pending matter. This is the correct answer. A lawyer shall not seek to influence a judge by means prohibited by law. Model Rule 3.5(a). Here, the attorney used her public position to attempt to influence the judge. Specifically, she noted that the uncle was the type of solid citizen and influential person who could help garner support for the budget. Clearly, this was an attempt to have the judge rule in the uncle's favor in the pending divorce.

An attorney represented a man in a case set for a jury trial. After the list of potential jurors was made available, the attorney hired a private investigator to interview the potential jurors and their family members concerning their relevant past experiences related to the subject matter of the action. The investigator did not inform the jurors or their family members that he was working on behalf of the attorney. The interviews were entirely voluntary and were not harassing. The attorney did not provide the report of the interviews to opposing counsel. He used the report to make decisions regarding jury selection. Is the attorney subject to discipline? a. Yes, because the attorney did not provide the report of the interviews to opposing counsel. b. No, because the interviews were entirely voluntary and not harassing. c. Yes, because the investigator, at the attorney's direction, communicated with potential jurors prior to trial. d. Yes, because the investigator did not inform the jurors or their family members that he was working on behalf of the attorney.

c. Yes, because the investigator, at the attorney's direction, communicated with potential jurors prior to trial. This is the correct answer. A lawyer shall not communicate ex parte with a judge, juror, prospective juror, or other official during the proceeding unless authorized to do so by law or court order. Model Rule 3.5(b). Here, the attorney violated the rule by using his investigator to communicate with potential jurors prior to trial.

A law firm has 300 lawyers in 10 states. It has placed the supervision of all routine administrative and financial matters in the hands of a nonlawyer. The nonlawyer is paid a regular monthly salary and a year-end bonus of 1 percent of the law firm's net income from fees. Organizationally, the non-lawyer reports to an attorney who acts as the managing partner of the law firm. The attorney deals with all issues related to the law firm's supervision of the practice of law. The nonlawyer does not have access to client files and does not control the professional judgment of lawyers in the firm. Is it proper for the attorney to participate in the law firm's use of the nonlawyer's services in this fashion? a. No, because the law firm is sharing legal fees with a nonlawyer. b. Yes, because the nonlawyer does not have access to client files. c. Yes, because the nonlawyer does not control the professional judgment of the lawyers in the firm. d. No, because the law firm is assisting a nonlawyer in the unauthorized practice of law.

c. Yes, because the nonlawyer does not control the professional judgment of the lawyers in the firm. This is correct. "A lawyer shall not practice with or in the form of a professional corporation or association authorized to practice law for a profit, if . . . a nonlawyer has the right to direct or control the professional judgment of a lawyer." Model Rule 5.4(d)(3). Here, the nonlawyer only supervises routine administrative and financial matters and does not control the professional judgment of lawyers in the firm. Importantly, this Rule's limitations "are to protect the lawyer's professional independence of judgement." Model Rule 5.4, Comment [1]. Consequently, this arrangement is proper under the Rules.

Last year, attorney Mira represented Sally, the owner of a small pizza restaurant, in a suit against Rome's Own, Inc., a supplier of shredded cheese, based on breach of contract. The parties settled the claim for $15,000. Mira and Sally remained friendly after the matter was settled, and have gone to the movies and to dinner a few times in the last year. Earlier today, Sally's next-door neighbor, Jose, came into Mira's office for a consultation and asked Mira to represent him in a civil suit against Sally. Jose alleges that Sally's pet macaw bit him, causing an infection and permanent scarring. Jose wants to get a court order against Sally requiring that the bird to be euthanized pursuant to a state law. Sally already paid for his medical care. Jose does not want further damages. Must Mira obtain Jose's informed consent before she can agree to represent Jose? a. No, because Jose's interests are not materially adverse to Sally's. b. No, because Jose's lawsuit is not substantially related to the breach of contract lawsuit against Rome's Own, Inc. c. Yes, because there is a significant rest risk that Mira's representation of Jose will be materially limited by her loyalty to Sally. d. Yes, because Jose's interests are directly adverse to Sally's.

c. Yes, because there is a significant rest risk that Mira's representation of Jose will be materially limited by her loyalty to Sally. This is the correct answer. Because Jose would become Mira's new client, Rule 1.7 applies. Under Rule 1.7(a)(2), a lawyer must obtain the consent of a prospective client if "there is a significant risk that the representation of [that client] will be materially limited by the lawyer's responsibilities to...a former client." Mira and Sally had a professional relationship and have had continuing social contact, so it is possible that Mira would be less vigorous in her advocacy on behalf of Jose because of her relationship to Sally.

Four years ago, an attorney represented a husband and wife, both high school teachers, in the purchase of a new home. Since then, the attorney prepared their tax returns and drafted their wills. Recently, the husband called the attorney and told her that he and his wife had decided to divorce, but wanted the matter to be resolved amicably. The husband stated that they were planning to file and process their own divorce case, utilizing the state's new streamlined divorce procedure, applicable in "no-fault" cases where there are no minor children. The husband asked if the attorney would agree to work with them to prepare a financial settlement that could be presented to the divorce court, reminding the attorney that the couple's assets were modest and that they wanted to "split it all down the middle." After considering the risks of a conflict of interest arising in this limited representation, the attorney wrote to the couple separately, and advise each that he or she might be better off with separate lawyers, but that the attorney would assist with the financial settlement agreement, charge an hourly fee of $140 per hour, provided they were in complete agreement and remained so. The attorney advised that if a conflict developed, or if either party was dissatisfied or uncomfortable about continuing with joint representation, the attorney would withdraw and would not represent either party from that point forward, forcing them to start all over again with separate lawyers. Finally, the attorney cautioned the husband and wife that the attorney would be representing both of them equally, would not and could not favor one or the other, and that their separate communications to her could not be kept confidential from the other party. Both the husband and the wife signed their individual copy of the letter, consenting to the joint representation, and returned them to the attorney. Was it proper for the attorney to accept the representation on these terms? a. No, because the attorney did not advise both the husband and the wife, in writing, that they should seek independent counsel before agreeing to enter into the financial settlement on the terms proposed. b. Yes, because the attorney had previously represented the husband and the wife in their joint affairs. c. Yes, because there was little risk that the interests of either the husband or the wife would be materially prejudiced if no settlement was reached. d. No, because the attorney conditioned representation upon receiving a waiver of client confidentiality.

c. Yes, because there was little risk that the interests of either the husband or the wife would be materially prejudiced if no settlement was reached.

A law firm, a professional corporation with five lawyer shareholders, employs 25 additional lawyers. Which of the following is proper? a. The firm's accountant, who is not a member of the bar, is a shareholder. b. The office manager, who is not a member of the bar, is executive vice president nof the law firm. c. A widow whose husband was a lawyer shareholder in the firm until he died five years ago continues to hold her husband's shares in the firm. d. Employees who are members of the bar are not made shareholders until they have been with the law firm for 10 years.

d. Employees who are members of the bar are not made shareholders until they have been with the law firm for 10 years. This is correct. There is nothing in the Rules that requires lawyers in a firm to be made shareholders within a certain amount of time.

An attorney was retained by a client to represent him in a paternity suit. The client's aunt believed the suit was unfounded and motivated by malice. The client's aunt sent the attorney a check for $1,000 and asked the attorney to apply it to the payment of the client's fee. The client's aunt told the attorney not to tell the client of the payment because the client "is too proud to accept gifts, but I know he really needs the money." Is it proper for the attorney to accept the aunt's check? a. Yes, because the attorney's charges to the client are reduced accordingly. b. No, because the client's aunt is attempting to finance litigation to which she is not a party. c. Yes, because the client's aunt is not attempting to influence the attorney's conduct of the case. d. No, because the attorney is not informing the client and obtaining the client's consent to retain the payment.

d. No, because the attorney is not informing the client and obtaining the client's consent to retain the payment. This is the correct answer. "A lawyer shall not accept compensation for representing a client from one other than the client unless: (1) the client gives informed consent." Model Rule 1.8(f)(1). Consequently, the attorney cannot accept the aunt's check without informing the client.

An attorney filed a personal injury suit on behalf of a plaintiff. The defendant was personally served with process. The attorney knows that the defendant is insured by an insurance company and that another attorney has been retained by the insurance company to represent the defendant. No responsive pleading has been filed on behalf of the defendant, and the time for filing expired over 10 days ago. Is the attorney subject to discipline if the attorney proceeds to have a default judgment entered? a. No, because any judgement will be satisfied by the insurance company. b. Yes, because the attorney failed to extend professional courtesy to another lawyer. c. Yes, because the attorney knows that another attorney had been retained by the insurance company to represent the defendant. d. No, because the attorney is properly representing her client's interests.

d. No, because the attorney is properly representing her client's interests. This is the correct answer. Model Rule 3.2 directs a lawyer to make reasonable efforts to expedite litigation consistent with the interests of the client. Model Rule 1.3 directs a lawyer to act with reasonable diligence and promptness in representing a client. Model Rule 3.4 covers fairness to opposing counsel, specifically issues such as falsifying evidence and inducing witnesses to give false testimony. Even though the attorney knows the insurance company is represented by another attorney, nothing in Model Rule 3.4 states that the attorney must warn the opposing attorney of a possible default judgment. Importantly, such a requirement could force a lawyer to violate Model Rules 3.2 and 1.3.

An attorney was hired by a restaurant owner. The owner asked the attorney whether any restaurants or bars had been prosecuted for violating a local law that stated alcohol could not be served before noon on Sundays. He also asked whether the law's $50 fine could be imposed for every sale on a Sunday before noon. The attorney accurately told the client that the fine could only be imposed once for each Sunday, not for each transaction, and that no one had been prosecuted under the law. Several weeks later, the attorney learned that the owner's restaurant had begun to have "Sunday booze breakfasts" where alcohol was served starting at 9 a.m. on Sundays. Is the attorney subject to discipline? a. Yes, because the attorney reasonably should have known that the information she gave the client would encourage him to violate the law. b. No, because the attorney was not acting as an officer of the court. c. Yes, because the attorney aided the owner in violating the law. d. No, because the attorney merely told the owner factual information regarding the law.

d. No, because the attorney merely told the owner factual information regarding the law. This is the correct answer. Although a lawyer is not allowed to counsel or assist a client in criminal or fraudulent conduct, a lawyer may help a client determine the validity, scope, meaning, or application of the law to a client's particular situation. Model Rule 1.2(d). Here, the attorney is not helping the owner break the law; she is simply stating facts regarding the law's fine and enforcement.

An attorney represented a seller in negotiating the sale of his ice cream parlor. The seller told the attorney in confidence that, although the business had once been very profitable, recent profits had been stable but modest. As the negotiations proceeded, the buyer appeared to be losing interest in the deal. Hoping to restore the buyer's interest, the attorney stated, "The ice cream business is every American's dream: happy kids, steady profits, and a clear conscience." The buyer bought the ice cream parlor but was disappointed when his own profits proved to be modest. Is the attorney subject to discipline? a. Yes, because the attorney made a false statement of fact to the buyer. b. Yes, because the attorney exaggerated the profitability of the business. c. No, because the attorney represented the seller, not the buyer. d. No, because the attorney's statement constitutes acceptable puffing in negotiations.

d. No, because the attorney's statement constitutes acceptable puffing in negotiations. This is the correct answer. A lawyer is prohibited from knowingly making a false statement of material fact or law to a third person in the course of representing a client. Model Rule 4.1. Omissions that rise to the level of an affirmative false statement are also improper. Importantly, the Rule applies on ly to statements of material fact. Under generally accepted standards of negotiation, certain types of statements ordinarily are not taken as statements of material fact. Model Rule 4.1, Comment [2]. Estimates of price, value, or settlement maximums are ordinarily placed in this category, and are considered mere "puffing" on the part of the attorney. Here, the attorney's comments as to "steady profits" amounted to mere "puffing."

An attorney represents a client who is under indictment for homicide. In the course of the representation, the client told the attorney that she had previously killed two other people. These murders are completely unrelated to the murder indictment for which the attorney is providing representation. With the client's consent, the attorney made a tape recording of the client's confession regarding the unrelated homicides. At the attorney's request, the client also drew a map of the remote locations of the victims' graves from the unrelated killings. Those bodies have not been found by the police, and the client is not a suspect in either crime, both of which remain unsolved. Is the attorney subject to discipline if he fails to voluntarily disclose to the authorities his knowledge of the two prior murders and the locations of the victim's bodies? a. Yes, because the attorney is impeding the state's access to significant evidence b. Yes, because as an officer of the court, the attorney must disclose any knowledge that he has, whether privileged or not, concerning the commission of the prior crimes by his client. c. No, because the attorney did not represent or advise his client with respect to the prior crimes. d. No, because the information was obtained by the attorney in the course of the representation.

d. No, because the information was obtained by the attorney in the course of the representation. A lawyer shall not reveal any information relating to the representation of a client unless the client gives informed consent. Model Rule 1.6(a). This is probably the most important Rule in the Model Rules of Professional Conduct. It is intended to promote candid communications between the client and attorney, so all the aspects of a particular legal problem become known and the lawyer can provide the most competent legal advice. Importantly, if a client knew the attorney could share his or her confidences to third parties, competent representation would become much more difficult, if not impossible, as the client would likely leave the lawyer in the dark on many important facts. Here, although the prior murders are not part of the present case, this information was obtained during the course of the attorney's representation, and consequently "relates" to that representation. Additionally, the prior murders would likely affect the attorney's strategy in the present case, specifically whether his client should plead guilty or testify at trial

Five years ago, an attorney represented a seller in the sale of the seller's home. The attorney has not represented the seller since that time. Recently, the attorney was approached by the seller's partner in a venture capital company formed two years ago. The partner and seller have agreed to dissolve their partnership, but cannot agree on the terms of the dissolution. The partner asked the attorney to sue the seller for an accounting of the partnership assets. If the attorney accepts the representation, is the attorney subject to disqualification? a. Yes, because there is no indication that at the time of the sale of the seller's home, the seller agreed that the attorney would not subsequently be precluded from representing other clients in suits against the seller. b. Yes, because the representation is directly adverse to the seller. c. No, because the seller did not sell the home while in the partnership. d. No, because the partnership dissolution is unrelated to the sale of the seller's home.

d. No, because the partnership dissolution is unrelated to the sale of the seller's home. This is the correct answer. According to Model Rule 1.9(a), "[a] lawyer who has formerly represented a client in a matter shall not thereafter represent another person in the same or a substantially related matter in which that person's interests are materially adverse to the interests of the former client." Here, the important point is that the partnership dissolution and the sale of the home are completely unrelated, so the attorney may accept the representation.

An attorney represents a famous politician in an action against a newspaper for libel. The case has attracted much publicity, and a jury trial has been demanded. After one of the pretrial hearings, as the attorney left the courthouse, news reporters interviewed the attorney. In responding to questions, the attorney revealed the identity of the reporter. Is the attorney subject to discipline for making this statement? a. No, because the trial has not commenced. b. Yes, because prospective jurors might learn of the attorney's remarks. c. Yes, because the attorney identified a prospective witness in the case. d. No, because the statement relates to a matter of public record.

d. No, because the statement relates to a matter of public record. This is the correct answer. During representation, a lawyer "shall not make an extrajudicial statement that the lawyer knows or reasonably should know will be disseminated by means of public communication and will have a substantial likelihood of materially prejudicing an adjudicative proceeding in the matter." Model Rule 3.6(a). However, there is an exception for "information contained in a public record." Model Rule 3.6(b)(2). Here, since the politician has sued the newspaper for libel, the identity of the reporter is a matter of public record.

The police arrested a middle school band teacher for repeatedly sexually abusing numerous female students aged 9 to 12. The molestation occurred during private music lessons and took place over a seven-year period before the teacher was arrested. Some of the victims told the police that they had reported the abuse to the school principal after it occurred, but the principal had failed to take action against the teacher. After the teacher was arrested, some of the victims and their families filed suit against the principal and the school board. The school board hired the firm of Schoenholtz & Koplow "to investigate the response of the school administration to allegations of sexual abuse of students." The board explained that it would need a full report so that it could decide whether to settle or litigate the claims against it. Meanwhile, the school board also hired a respected trial lawyer, Lindsay, at another firm, to represent the board in the lawsuit. Schoenholtz & Koplow interviewed all the officers of the school board to find out what they knew about the molestation, when they learned what they knew, and what they did after they receive the information. Their notes and report were turned over to the school board, which gave them to Lindsay. The plaintiffs in the lawsuit requested, during discovery. the notes made by Schoenholtz & Koplow during their interviews. Lindsay must: a. Turn over the notes to the plaintiffs because while the communication may be protected, the underlying facts are not protected from disclosure. b. Turn over the notes to the plaintiffs because Schoenholtz & Koplow was hired to do the investigation, not to defend the school board against the lawsuit. c. Refuse to turn over the notes to the plaintiffs, based on attorney-client privilege, because the crimes discussed in those documents could result in liability for the school board. d. Refuse to turn over the notes to the plaintiffs because they are protected by attorney-client privilege.

d. Refuse to turn over the notes to the plaintiffs because they are protected by attorney-client privilege. As in the Upjohn case, 449 U.S. 383 (1981), the lawyer is collecting information for the client so the client can mount a legal defense. See also MR 1.13 ("Paragraph (c) [on revealing potential wrongdoing within the organization] shall not apply with respect to information relating to a lawyer's representation of an organization to investigate an alleged violation of law, or to defend the organization or an officer, employee or other constituent associated with the organization against a claim arising out of an alleged violation of law"). Further, it is irrelevant that a different law firm is representing the client in the lawsuit

An attorney is a member of the bar in one state and is also licensed as a stockbroker in another. In his application for renewal of his stockbroker's license the attorney knowingly filed a false financial statement. Soon thereafter, the attorney is convicted in the second state for knowingly filing a false financial statement. Is the attorney subject to discipline as a member of the bar of the first state? a. Yes, because he was first convicted of a criminal offense in the second state. b. No, because his action was not done in his capacity as an attorney. c. No, because his action was not in the first state. d. Yes, because his actions involved dishonesty or misrepresentation.

d. Yes, because his actions involved dishonesty or misrepresentation. This is the correct answer. Model Rule 8.4 defines the acts that constitute misconduct by a lawyer. Model Rule 8.4(c) describes as professional misconduct any conduct "involving dishonesty, fraud, deceit or misrepresentation." The definition is not limited to misconduct in connection with the practice of law but, under Comment [2], extends to all offenses that "indicate lack of those characteristics relevant to law practice," including violence, dishonesty, and breach of trust. Also, misconduct is misconduct wherever it occurs - whether in the state in which the attorney is licensed to practice or elsewhere. Here, the attorney has knowingly filed a false financial statement in connection with a professional license application, a clear violation of the Rule.

Herman was mugged in a dark alley, and his wallet, which contained $200 in $20 bills, was stolen. Moments after the assailant fled with his money, Herman spotted a police officer and reported the crime. Herman described the assailant as a male who had dark hair, was of medium height, and wore a dark colored shirt and white sneakers. He said the assailant had showed him a knife. The officer alerted other officers in the vicinity by radio. An officer arrested Juan three blocks away. Juan has dark hair and is of medium height. He was wearing a dark blue shirt and white sneakers. When searched, he was found to have $454 in cash in his pocket, including eight twenty dollar bills. He was not in possession of a knife. When arrested, Juan acted suspiciously and refused to answer any questions. He has not confessed. Herman's wallet was never found, and no knife was found in the vicinity. The case was assigned to Marcella, a prosecutor. Marcella investigates but discovers no additional facts. She believes that Juan is guilty, though she doubts that a jury would find him guilty beyond a reasonable doubt. In fact, she is uncertain whether the charge is even supported by probably cause. Is it proper for Marcella to charge Juan with robbery? a. No, if she thinks that he is not guilty beyond a reasonable doubt. b. Yes, because she believes that Juan is guilty. c. No, because she doubts that a jury would convict him. d. Yes, because she does not know that the charge is not supported by probable cause.

d. Yes, because she does not know that the charge is not supported by probable cause. This is correct. Rule 3.8(a) bars a prosecutor from moving forward with a charge that the prosecutor "knows" is not supported by probable cause. "Knows" "denotes actual knowledge of the fact in questions." Rule 1.0(f). Marcella is uncertain about whether the probable cause standard is met, so she does not "know" that the charge would be unsupported by probable cause.

An attorney represented a buyer in a real estate transaction. Due to the attorney's negligence in drafting the purchase agreement, the buyer was required to pay for a survey that should have been paid by the seller. The attorney fully disclosed this negligence to the buyer, and the buyer suggested that he would be satisfied if the attorney simply reimbursed the buyer for the entire cost of the survey. Although the buyer might have recovered additional damages if a malpractice action were filed, the attorney reasonably believed that the proposed settlement was fair to the buyer. He sent the buyer a letter stating that the buyer should consider seeking independent representation before making a decision. Then, in order to forestall a malpractice action, the attorney readily agreed to make the reimbursement. The attorney drafted a settlement agreement, and it was executed by both the attorney and the buyer. Was the attorney's conduct a proper? a. No, because the attorney settle the case involving liability for malpractice while the matter was still ongoing. b. Yes, because the attorney reasonably believed that the proposed settlement was fair to the buyer. c. No, because the buyer was not separately represented in negotiating and finalizing the settlement agreement. d. Yes, because the attorney advised the buyer in writing that the buyer should seek independent representation before deciding to enter into the settlement agreement.

d. Yes, because the attorney advised the buyer in writing that the buyer should seek independent representation before deciding to enter into the settlement agreement. In settling a claim or potential claim for malpractice, a lawyer must advise an unrepresented client in writing of the desirability of seeking independent legal counsel. Model Rule 1.8(h)(2).

An attorney represented a client in an action against the client's former partner to recover damages for breach of contract. During the representation, the client presented the attorney with incontrovertible proof that the former partner had committed perjury in a prior action that was resolved in the partner's favor. Neither the attorney nor the client was involved in any way in the prior action. The attorney believes that it would be detrimental to the client's best interests to reveal the perjury because of the implication that might be drawn from the former close personal and business relationship between the client and the former partner. Would it be proper for the attorney to fail to disclose the perjury to the tribunal? a. No, because the attorney has knowledge that the former partner perpetrated a fraud on the tribunal. b. No, because the information is unprivileged. c. Yes, because neither the client nor the attorney was involved in the prior action. d. Yes, because the attorney believes that the disclosure would be detrimental to the client's best interests.

d. Yes, because the attorney believes that the disclosure would be detrimental to the client's best interests. The information regarding the perjury committed by the client's former partner is "information relating 'to the representation" of the client, which generally may not be disclosed without client consent. Model Rule 1.6(a). Confidential information may be disclosed without client consent under Model Rule 1.6(a) when "the disclosure is impliedly authorized in order to carry out the representation," but there would be no implied authorization where, as here, the disclosure would likely harm the client.

An attorney hired a recent law school graduate as an associate. For the first six months, the associate was assigned to draft legal documents that the attorney carefully reviewed and revised before filing. However, shortly after the associate was admitted to the bar, the attorney told the associate that he would be going on vacation the following week and was assigning her the representation of the landlord in a housing case that was going to trial while he was away. The associate had never conducted or observed a trial before and, because she had not previously worked on any housing cases, she was unfamiliar with the relevant law and procedure. She did not believe that she would have enough time to learn everything that she needed to know, but she was reluctant to decline the assignment. Before the trial began, she met with the landlord and disclosed that this would be her first trial, but the landlord did not object. Although the associate prepared diligently, the landlord lost the trial. Is the attorney subject to discipline? a. No, because the landlord did not object to the associate's representation. b. No, because the attorney could reasonably assume that, having been admitted to the bar, the associate was capable of conducting the trial. c. Yes, because the landlord lost at trial. d. Yes, because the attorney did not ensure that the associate was competent to conduct the trial on her own.

d. Yes, because the attorney did not ensure that the associate was competent to conduct the trial on her own. This is the correct answer. A newly admitted lawyer in a firm must have his or her work properly supervised by a more experienced lawyer to ensure that all lawyers in the firm conform to the Model Rules of Professional Conduct. Model Rule 5.1. Under Model Rule 1.1, a law firm owes a client a duty of competence, specifically the appropriate "legal knowledge, skill, thoroughness and preparation reasonably necessary for the representation." Consequently, when a firm uses less-experienced lawyers to perform client work, the partners must supervise that work. In this case, the associate, who had never conducted or observed a trial and had not worked on landlord-tenant cases before, needed supervision. There was not enough time for the associate to become familiar with the relevant law and procedure, nor did she have the experience to competently handle the case. Importantly, the attorney did not take the proper precautions to make sure that the associate was adequately prepared to carry out the assignment.

An attorney represented a client in negotiating a large real estate transaction. The buyer who purchased the real estate from the client has filed suit against both the client and the attorney, alleging fraud and violation of the state unfair trade practices statute. The attorney had advised the client by letter against making the statements that are the basis for the buyer's claim. The attorney and the client are each represented by separate counsel. In responding to a deposition under subpoena, the attorney wishes to reveal, to the extent the attorney reasonably believes necessary to defend herself, confidential information imparted to the attorney by the client that will be favorable to the attorney but damaging to the client. The client did not object to the disclosure. At this point, no criminal charges have been brought against the attorney. Is it proper for the attorney to reveal such information? a. No, because the disclosure will be detrimental to the client. b. Yes, because the client did not object to the disclosure. c. No, because criminal charges have not also been brought against the attorney d. Yes, because the attorney may reveal such information to defend herself against a civil claim.

d. Yes, because the attorney may reveal such information to defend herself against a civil claim. A lawyer may reveal confidential information "to establish a defense to a criminal charge or civil claim against the lawyer based upon conduct in which the client was involved." Model Rule 1.6(b)(5). Here, since the buyer is suing both the client and attorney over the same real estate transaction, the attorney may reveal the information.

An attorney, who is corporate counsel for a company, is investigating a possible theft ring in the parts department of the company. The attorney knows that an employee has worked in the parts department a long time and believes that the employee is a suspect in the thefts. The attorney believes that if the employee were questioned, the employee would not answer truthfully if she knew the real purpose of the questions. The attorney plans to question the employee and falsely tell her that she is not a suspect and that her answers to the questions will be held in confidence. Is the attorney subject to discipline if she so questions the employee? a. No, because no legal proceedings are now pending. b.Yes, because the attorney did not first advise the employee to obtain counsel. c. No, because the attorney did not give legal advice to the employee. d. Yes, because the attorney's conduct involves misrepresentation.

d. Yes, because the attorney's conduct involves misrepresentation. This is the correct answer. A lawyer may not "engage in conduct involving dishonesty, fraud, deceit, or misrepresentation." Model Rule 8.4(c). Here, the attorney is planning to falsely tell the employee she is not a suspect and that her answers will be held in confidence. This is a clear violation of the Rules.

A client retained an attorney to recover for a personal injury. In the retainer agreement signed by the client and the attorney, the client agreed to cooperate fully and pay the attorney a contingent fee computed as a percentage of the amount of recovery after expenses: 25 percent if settled before trial, 30 percent if settled before verdict, 35 percent after verdict, and 40 percent after appeal. The attorney's representation of the client in the matter extended over a three-year period, during which the attorney advanced a large amount for litigation expenses. After trial, the client obtained a jury verdict for an amount larger than either the attorney or the client had anticipated. However, the defendant filed an appeal based on questions of evidence and the measure of damages. Meanwhile, the defendant made an offer of settlement for approximately the amount the attorney had originally projected as reasonable to expect. The client, who was hard-pressed financially, directed the attorney to accept the offer and settle. The attorney refused, because she was confident that there was no reversible error in the trial and that the appeal was without merit. The attorney reasonably believed that the appeal was filed solely to gain negotiating advantage in settlement negotiations. Is the attorney subject to discipline? a. Yes, because the attorney's percentage under the fee contract increased after appeal. b. No, because evaluation of the merits of an appeal requires the exercise of independent professional judgment. c. No because the decision whether to settle or defend an appeal is a tactical matter for the attorney to determine. d. Yes, because the client directed the attorney to accept the settlement offer.

d. Yes, because the client directed the attorney to accept the settlement offer. Subject to prior limitations on the scope of representation and the lawyer's duty not to assist a client in criminal conduct, a lawyer "shall abide by a client's decisions concerning the objectives of representation.'' Model Rule 1.2(a). Specifically, a lawyer must abide by a client's decision to settle a matter, or, in a criminal case, the client's decision as to the "plea to be entered, whether to waive jury trial, and whether the client will testify." Model Rule 1.2(a). However, the lawyer has implied authorization to decide what actions are required to achieve the client's goals, as long as the lawyer consults with the client. Model Rule 1.4(a)(2). Here, once the client decided to settle, the attorney was required to accept the offer, whether or not the attorney reasonably believed the appeal was filed to gain an advantage in the settlement negotiations.

Tandy is a practicing lawyer and adjunct law professor at Frostburg Law School, which is part of Frostburg University. Two Caucasian high school seniors have asked her to represent them in their lawsuit against Frostburg University. They allege that Frostburg University did not admit them to the undergraduate program because of race discrimination. Specifically, they allege that students of color with similar test scores and grades were admitted to Frostburg University, while they were not. Tandy reasonably believes that she can competently and diligently represent the students in their lawsuit. She obtains informed consent, confirmed in writing, from both the students and the university. May Tandy represent the two students in their lawsuit? a. Yes, because the university gave informed consent, confirmed in writing. b. No, because the interests of the students are directly adverse to the interests of the university. c. No, because the university employs Tandy, so this representation would present a non-consentable conflict of interest for Tandy. d. Yes, because the students gave informed consent, confirmed in writing.

d. Yes, because the students gave informed consent, confirmed in writing. The conflict is a "material limitation" conflict under Rule 1.7(a)(2). The students' consent is required because there is a significant risk that Tandy's representation of them will be materially limited by her obligations to her employer. For example, she might advocate less vigorously on the students' behalf because the university might terminate her position in reaction to her "biting the hand that feeds her."

A corporation hired a law firm to handle all of its corporate work. The firm had not previously represented a corporation on an ongoing basis, but decided that it wanted to attract additional corporate clients. Accordingly, the partners handling the corporation's work began a practice of giving to all lawyers in the firm, on a monthly basis, detailed descriptions of the work they were dong for the corporation for the purpose of illustrating what the firm could do for corporate clients. One of the partners mentioned this practice to the corporation's management, and the corporation complained that its confidences had been violate. Was it proper for the partners to give detailed descriptions of the work being done for the corporation to other lawyers in the firm? a. No, because the attorney must follow the client's instructions. b. No, because the client did not consent to the attorney's withdrawal. c. Yes, because lawyers may discuss client information with other lawyers in the firm, regardless of client instructions to the contrary, so long as the disclosure does not disadvantage the client. d. Yes, because, absent client instructions to the contrary, layers may discuss client information with other lawyers in the firm.

d. Yes, because, absent client instructions to the contrary, layers may discuss client information with other lawyers in the firm. "Except to the extent that the client's instructions or special circumstances limit that authority, a lawyer is impliedly authorized to make disclosures about a client when appropriate in carrying out the representation . . . . Lawyers in a firm may, in the course of the firm's practice, disclose to each other information relating to a client of the firm, unless the client has instructed that particular information be confined to specified lawyers." Model Rule 1.6, Comment [5]. Consequently, it was proper for the partners to give detailed descriptions of the work being done.


Conjuntos de estudio relacionados

Management: Information Systems 410 CHS 9-12

View Set

Pronouns: Subjects, Direct Objects, Indirect Objects, Predicate Nominatives, and Objects of the Preposition.

View Set

Lesson 4: Understanding Capacitance and How it Affects a Circuit

View Set

Most Missed Volleyball questions - Referee

View Set

1984 by George Orwell -- Characters

View Set

Variación sintáctica entre OD y OI

View Set

Chapter 39 Humidity and Bland Aerosol Therapy

View Set